PDA

توجه ! این یک نسخه آرشیو شده میباشد و در این حالت شما عکسی را مشاهده نمیکنید برای مشاهده کامل متن و عکسها بر روی لینک مقابل کلیک کنید : رفع اشكال المپیاد نجوم



صفحه ها : [1] 2 3

پیام بهرام پور
09-13-2010, 06:11 PM
دوستان می توانند در این تاپیک سوالات و اشكالات المپیادی خود را بپرسند.

ان شاالله در اسرع وقت به آنها پاسخ خواهيم داد.


http://www.alexisdawes.com/QuestionMarkFace.jpg (http://www.alexisdawes.com/QuestionMarkFace.jpg)

AslanAslan
12-27-2010, 04:43 PM
برای شروع من سوالی طرح می کنم. امیدوارم با بحث کردن این تاپیک ادامه پیدا کنه.


http://www.forexrainbow.com/files/uqt0093u5v9lask787qv.jpg

این تصویر را در تاپیک ببین و بگو جناب Mostafa لطف کرده و گذاشته بودن. چه اطلاعاتی می توان از این عکس استخراج کرد؟ در صورت تمایل می توانید اندازه گیری کنید و گزارش کنید.
به عنوان یک مثال ساده می شود عرض جغرافیایی محل عکس برداری را تعیین کرد.
رو این مساله فکر کنین. وسایل مورد نیاز هم فقط کاغذ - خودکار - ماشین حساب - خط کش و نقاله است!

169
12-27-2010, 06:34 PM
آقای نور قاسمی اگه حساب بشه عرضی که به دست میاریم دقیق هستش؟!

AslanAslan
12-27-2010, 07:26 PM
بله. البته مهم اینه که شما منظورتون از دقیق چقدر هست. بازه خطا فکر کنم در حد ۵ درجه باشه. نمی دونم چقدر آمار و احتمال بلد هستین (اگر دانشگاهی باشین که حتما درسشو خوندین) می شه خطای اندازه گیریمون رو محاسبه کرد.
با فرض این که در اندازه گیری خطا نداریم, جواب ما دقیق دقیق خواهد بود.

پیام بهرام پور
12-28-2010, 09:50 PM
بهتر نیست اول یه توضیحی راجع به این عکس انالما بفرمایید؟
خیلی از دوستان نمی دانند آنالما چی هست اصلاً!
درضمن اگر می بینید که دوستان المپیادی کم سر می زنند به این قسمت به خاطر این است که الان در حال امتحانات دی ماهشان هستند

someone
01-08-2011, 06:16 PM
سلام
میشه لطفا یکم در مورد تعدیل زمان و خورشید میانگین یکم توضیح بدین
مرسی.

Astronomer
01-09-2011, 11:39 PM
سلام
میشه لطفا یکم در مورد تعدیل زمان و خورشید میانگین یکم توضیح بدین
مرسی.
عليك السلام!!!
اين قسمت بسيار شيريني از زمان هست كه به نظر من بهترين توضيح اون رو كتاب "ستاره شناسي: اصول و عمل" داده! توجه كنيد:

در گذشته بدون توجه به فصل مردم از ساعت آفتابي براي تعيين وقت استفاده ميكردند! بدين شكل كه هر شبانه روز 24 ساعت و هر روز 12 ساعت!!!
به اين ترتيب جز در اعتدالين در مابقي روزهاي سال ساعت تعريف شده در روز با شب متفاوت بود! مثلا چون شبهاي زمستان بلندتر از روزهاي آن اند، 1ساعت در شب طولانيتر از 1ساعت در روز بود!!! اين را هم ميدانيم كه در طي يكسال همه شبانه روزها با هم برابر نيستند و يك شبانه روز در زماني كه زمين در حضيض است بلندتر از يك شبانه روز زمين در اوج مداري است!!!
(ميدونيد چرا؟ سعي كنيد تحليل كنيد! اگر نشد بگيد تا بگم!) تقريبا در بين زمستان زمين در حضيض قرار ميگيرد!

حال ما يك خورشيد تعريف ميكنيم كه 1) روي استوا طي مسير ميكند(نه روي دايرةالبروج) 2) سرعت زاويه اي اين خورشيد ثابت است و برابر سرعت زاويه اي ميانگين خورشيد واقعي در يكسال فرض مي شود!(يعني مدار اين جسم سماوي حول زمين دايره اي فرض مي شود نه بيضوي)

در اينجاست كه براي بدست آوردن ساعت دقيق به اختلاف بعد خورشيد واقعي و ميانگين نياز داريم چون ميتوانيم زاويه ساعتي خورشيد واقعي را از روي ساعت آفتابي بخونيم و ميدونيم كه اختلاف بعد دو جسم سماوي، قرينه اختلاف زاويه ساعتي آنهاست و زاويه ساعتي خورشيد ميانگين را بيابيم و ساعت محلي را بدست آوريم!!! ميدانيد كه يك مرحله ديگه كار تبديل ساعت محلي به ساعت منطقه اي است كه با دانستن اختلاف طول جغرافيايي بين خود و مرجع منطقه اين كار نيز آسان است!!!

تعديل زمان = بعد خورشيد ميانگين منهاي بعد خورشيد واقعي

اما اين تعديل زمان را از نموداري استخراج ميكنيم كه جالب است بدانيد در اول فروردين تعديل زمان را صفر نمي دهد!! چرا؟ چون اين نمودار از طريق زير بدست مي آيد:

1) زماني كه خورشيد در حضيض مداري است (1ژانويه) جسمي خيالي به نام خورشيد متوسط ديناميك همراه با خورشيد واقعي شروع به حركت روي دايرةالبروج ميكند. با اين تفاوت كه سرعت زاويه آن ثابت و برابر سرعت زاويه اي ميانگين خورشيد واقعي در يك سال است!!!
ميدانيد كه در اين زمان خورشيد واقعي از اين جسم خيالي پيشي ميگيرد و زودتر به نقطه اعتدال بهاري ميرسد! پس زماني بيشتر ميگذرد تا اين جسم خيالي به نقطه اعتدال بهاري برسد!

2) زماني كه خورشيد متوسط ديناميكي به اعتدال بهاري رسيد(خورشيد در ميل مثبت و جلوتر از آن است) خورشيد ميانگين روي استوا با شروطي كه گفتيم شروع به حركت ميكند.

خوب در همين لحظه اول كه چند روز از نوروز گذشته تعديل زماني منفي خواهيم داشت و پس از گذشت يك سال كه خورشيد واقعي به اعتدال بهاري ميرسد، خورشيد متوسط كمي عقبتر روي استوا در حال نزديك شدن به اعتدال بهاري است!!! پس در لحظه تحويل سال هم تعديل زماني منفي داريم!!!

انشاالله كه مفيد بوده باشد!!!

Setare KOchOlO
01-10-2011, 12:55 AM
حال ما يك خورشيد تعريف ميكنيم كه 1) روي استوا طي مسير ميكند(نه روي دايرةالبروج) 2) سرعت زاويه اي اين خورشيد ثابت است و برابر سرعت زاويه اي ميانگين خورشيد واقعي در يكسال فرض مي شود!(يعني مدار اين جسم سماوي حول زمين دايره اي فرض مي شود نه بيضوي)




سلام ...:yaeh am not durnk:
من اين تيكه كه نارنجي رنگش كردمو متوجه نشدم ...؟؟؟!!!! يعني يه مدار گردشي جديد؟؟؟اونم دايره اي شكل نه بيضوي؟؟؟؟چرا؟؟؟ بعد اين براي همه طول سال ساعت يكسان بدست نمياد؟؟؟

:slow:

Astronomer
01-10-2011, 01:59 AM
سلام ...:yaeh am not durnk:
من اين تيكه كه نارنجي رنگش كردمو متوجه نشدم ...؟؟؟!!!! يعني يه مدار گردشي جديد؟؟؟اونم دايره اي شكل نه بيضوي؟؟؟؟چرا؟؟؟ بعد اين براي همه طول سال ساعت يكسان بدست نمياد؟؟؟

:slow:
خوب همونطور كه از تعريف شبانه روز بر مي ياد، يه جسم سماوي كه دوبار از نصف النهار بگذره يه شبانه روز برمبناي اون جسم طي شده!!!

در شبانه روز نجومي از گذر ستاره ها استفاده ميشه!
در شبانه روز خورشيدي از خورشيد واقعي،
و در شبانه روز مدني قراردادي از خورشيد ميانگين(همونكه تعريفش كرديم)

بله ويژگي اين جسم سماوي اينه كه اگر شبانه روز نجومي رو ثابت فرض كنيم (گردش به دور زمين در يك روز رو ثابت فرض كنيم)، اختلاف شبانه روز نجومي و مدني همواره عددي ثابت است و طول شبانه روز مدني ثابت مي ماند كه اين براي امور مدني بسيار مهم است!!!

خوب اگر ميخوايد بدونيد چرا طول روز ثابت ميمونه، بنويسيد كه شبانه روز مدني ميشه شبانه روز نجومي به علاوه يه عددي كه بايد زمين بچرخه تا جبران پيش روي خورشيد متوسط توي مدار فرضي كه گفتيم بشه!!! ميبيند كه ثابت ميمونه!

اين دو شرط بيان گر اين است كه در تغيير طول شبانه روز در فصول متفاوت دو عامل مؤثرند: 1) ميل 23درجه و 27دقيقه اي استوا و دايرةالبروج 2) بيضوي بودن مدار زمين

باز هم ما در خدمتيم اگر سؤالي هست!

someone
01-10-2011, 06:27 PM
ممنون از توضیحتون:hope my fake smile
توی کتاب نجوم کروی قیاس های دالامپر و نپر و فرمول های هاور سینوس بدرد می خورن؟
دوما توی این همه سال که کتاب نجوم کروی چاپ شده کسی یادش نیفتاده که ماشین حساب مهندسی اختراع شده و تو دسترس مردم هست و نیازی به روش های لگاریتمی نیست؟

Astronomer
01-12-2011, 08:44 AM
خوب دوستان! در وبلاگ دوستان عزيزم ((گروه المپياد نجوم تبريز (http://forum.avastarco.com/forum/www.taog.blogsky.com/1387/09/02/post-61/))) تونستم يكي از بهترين نمودارهاي تعديل زمان رو پيدا كنم!

اول از همه اينكه اين نمودار از اعتدال بهاري شروع ميشه و اين خيلي خوبه! ((نيازي به تبديل تاريخ ها به ميلادي نداريد براي استفاده))

بعد هم ميتونيد مراحل مشخص شدن تعديل زمان رو ببينيد!

ببينيد:


http://forum.avastarco.com/forum/attachment.php?attachmentid=322&stc=1&d=1294808741


البته مطمئنم ميتونيد از راهنماي نمودار پي به مفهوم اين خطوط ببريد ولي من هم ذكر ميكنم:


سبز) تفاوت بعد خورشيد واقعي و خورشيد ميانگين ديناميكي كه پيشتر ذكر شد را نشان ميدهد!


آبي) تفاوت بعد خورشيد ميانگين با خورشيد ميانگين ديناميكي را نشان ميدهد!


زرشكي) معادله زمان يا همان تعديل زمان و به عبارتي ديگر اختلاف بعد خورشيد ميانگين و خورشيد واقعي را نشان مي دهد!


كه زرشكي ميشه جمع جبري آبي و سبز!
تعديل زمان = بعد خورشيد ميانگين - بعد خورشيد واقعي
تعديل زمان = (بعد خورشيد ميانگين - بعد خورشيد ميانگين ديناميكي) + (بعد خورشيد ميانگين ديناميكي - بعد خورشيد واقعي)

Astronomer
01-12-2011, 08:48 AM
ممنون از توضیحتون:hope my fake smile
توی کتاب نجوم کروی قیاس های دالامپر و نپر و فرمول های هاور سینوس بدرد می خورن؟
دوما توی این همه سال که کتاب نجوم کروی چاپ شده کسی یادش نیفتاده که ماشین حساب مهندسی اختراع شده و تو دسترس مردم هست و نیازی به روش های لگاریتمی نیست؟
خواهش ميشود!
به درد كه ميخورند ولي به درد المپياد نجوم نمي خورن!!!
خوب قضيه اين هست كه ناشر حق نداره در متن كتاب دست ببره!

كتاب نجوم كروي واقعا يك كتاب جامع بوده و هست و تا مدت ها خواهد بود! پس حذف جزئي از اون بدون اجازه مؤلف كاري انساني، اخلاقي و قانوني نيست!

Mehrnoosh
04-05-2011, 09:43 PM
http://www.irupload.ir/images/vh53pftb79ftoka6x2wo.jpg

اگه می شه برای قسمت اولش راهنمایی بکنید :help:

Ehsan
04-06-2011, 11:23 AM
http://www.irupload.ir/images/vh53pftb79ftoka6x2wo.jpg

اگه می شه برای قسمت اولش راهنمایی بکنید :help:
اگه موقعِ شروعِ حرکت از تهران متوجه شده باشه پس شما باید فاصله ی شخص رو از جایی که اون ستاره در سمت الرئس قرار داره رو بدونید. در لحظه ی شروع این فاصله 60=30-90 (ارتفاع-90) درجه است. اگر شما در هر لحظه فاصله تون رو از اون نقطه (جایی که ستاره ی مذکور در سمت الرئس قرار داره) رو بدونید و بعد اون رو نودِ منهاش بکنید ارتفاعِ ستاره به دست میاد. فقط باید دقت کنید به دلیلِ چرخشِ زمین اون نقطه ثابت نیست.

Mehrnoosh
04-07-2011, 12:35 AM
منظور از شعاع زاویه ای ماهواره چیه؟

Ehsan
04-07-2011, 08:24 AM
منظور از شعاع زاویه ای ماهواره چیه؟

حدس می زنم که منظور نصفِ قطرِ زاویه ایش هست.

Mehrnoosh
04-07-2011, 09:47 AM
حدس می زنم که منظور نصفِ قطرِ زاویه ایش هست.
http://www.irupload.ir/images/t152r37yqryki2dbyygi.jpg

مگه ماهواره هم قطر زاویه ای داره؟

Astronomer
04-07-2011, 02:55 PM
http://www.irupload.ir/images/t152r37yqryki2dbyygi.jpg

مگه ماهواره هم قطر زاویه ای داره؟
خوب وقتي شعاع ماهواره رو ميده و فاصله اون رو پس دقيقا منظورش نصف قطر زاويه اي هست ديگه!!!
در واقع تابعيت معكوس فاصله بر حسب زمان از شما خواسته شده!!

Mehrnoosh
04-07-2011, 03:46 PM
عجب گیجی شدما !!!!!!
مرسی!!!:thanks:

roset
04-08-2011, 05:18 PM
ببخشید دوستان مجموعه سوالات آقای چرتاب سلطانی رو چه طور میتونم پیدا کردم تو وبلاگ انجمن نجوم تبریز هست اما فیلتر شده میشه راهنمایی کنید

Mehrnoosh
04-08-2011, 10:56 PM
http://www.4shared.com/document/rRlM...chsoltani.html
http://www.4shared.com/document/0n-M...hSoltani2.html

M45
04-08-2011, 11:51 PM
رزت جان اگه دوست داری میخوای ایمیلتو بده تا من برات بفرستمش!

Mehrnoosh
04-10-2011, 07:32 PM
ببخشید دوستان مجموعه سوالات آقای چرتاب سلطانی رو چه طور میتونم پیدا کردم تو وبلاگ انجمن نجوم تبریز هست اما فیلتر شده میشه راهنمایی کنید

taog فیلتر نشده!!
www.taog.blogsky.com

Mehrnoosh
04-11-2011, 09:57 AM
میشه فرمول های محاسبه ی لیبراسیون و پهن شدگی طیف ستاره ها رو یه مروره کوتاه بکنید !!!!!

Mehrnoosh
04-12-2011, 09:34 AM
:stupido:
:help::help:

Mehrnoosh
04-16-2011, 01:14 PM
http://irupload.ir/images/mniir2ige785z5vnuns7.jpg

اینو به کمک انرژی و سرعت بیضی ؛تغییرات a بر حسب زمان رو بدست آوردم ولی بعدش رو نمی دونم چه کنم؟!!! :banghead:

roset
04-16-2011, 06:54 PM
به این سوال توجه کنید :در منظومه ای فرا خورشیدی سیاره ای به جرم مشتری حول ستاره ای از رشته اصلی با جرم 1.4 جرم خورشید میگردد.اگر شعاع مداری این سیاره مشتری گون 2.2*11^10 متر باشد و هر 14 ساعت یکبار به دور خودش بچرخد تعیین کنید اختلاف شبانه روز خورشیدی این سیاره با مشتری چقدر است؟جرم مشتری شعاع مداری و دوره تناوب چرخشی نجومی مشتری و جرم خورشید را داریم.
سوال به نظر ساده است اما طراح در حل سوال برای محاسبه ی t سیاره ای که در منظومه فرا خورشیدی قرار داره جرم خورشید رو در فرمول قرار داده .من فکر میکنم این انتخاب غلطه میشه توضیح بدید .

Ehsan
04-17-2011, 11:07 PM
http://irupload.ir/images/mniir2ige785z5vnuns7.jpg

اینو به کمک انرژی و سرعت بیضی ؛تغییرات a بر حسب زمان رو بدست آوردم ولی بعدش رو نمی دونم چه کنم؟!!! :banghead:

احتمالا باید جایی (زمانی ) رو پیدا کنی که نیم قطر اطول a برابر با شعاعِ زمین بشه چون کمتر نمیتونه بشه و الا به زمین برخورد می کنه. البته اطمینان ندارم! ولی حدس می زنم اگر طراح سوال چیزِ ساده ای تو ذهنش باشه این جوری حل میشه. در غیر این صورت راهِ دقیق تر (که فکر نمی کنم طراح چنین قصدی داشته باشه!) اینه که شما گشتاور حاصل از نیروی اصطکاک با جو و در نتیجه تغییرِ تکانه ی زاویه ای رو به دست بیارید! وقتی تغییرِ تکانه ی زاویه ای رو به دست آوردید نهایتا می تونید خروج از مرکز رو به دست بیارید. ولی این جوری خیلی پیچیده تر میشه




به این سوال توجه کنید :در منظومه ای فرا خورشیدی سیاره ای به جرم مشتری حول ستاره ای از رشته اصلی با جرم 1.4 جرم خورشید میگردد.اگر شعاع مداری این سیاره مشتری گون 2.2*11^10 متر باشد و هر 14 ساعت یکبار به دور خودش بچرخد تعیین کنید اختلاف شبانه روز خورشیدی این سیاره با مشتری چقدر است؟جرم مشتری شعاع مداری و دوره تناوب چرخشی نجومی مشتری و جرم خورشید را داریم.
سوال به نظر ساده است اما طراح در حل سوال برای محاسبه ی t سیاره ای که در منظومه فرا خورشیدی قرار داره جرم خورشید رو در فرمول قرار داده .من فکر میکنم این انتخاب غلطه میشه توضیح بدید .

احتمالا طراح اشتباه کرده! مشکلی نیست. از این چیز ها پیش میاد

Mehrnoosh
04-18-2011, 12:08 AM
http://irupload.ir/images/aogyyv7502getetvupb.jpg
http://irupload.ir/images/tdbihr698xgv58ap69lj.jpg
این پاسخ نامه ی خودشه
عکس دوم قسمتی که نوشته delta a/delta T به توان N ، رو دلیلشو نمی فهمم!!!

Fowad
04-18-2011, 10:36 PM
سلام و خداقوت:)
این روزها نزدیک امتحان مرحله ی دوم است و می دانم که بعدها این روزها برایتان خاطرات به یادماندنی ای خواهد بود...تلاش و زحمت و نشاط و امید و توکل و محبت و دعا و موفقیت و انرژی و شور و آینده و آرامش و ...کلماتی که این روزها زیاد می شنوید...ان شاالله بهترین درس ها را با این کلمات یاد بگیریم...:)
من با اجازه تان در جواب سوالات اخیر چیزی نوشته ام بسیار مختصر که در لینک های زیر است...چون تایپ فرمول و ...خیلی وقت گیر است باید ببخشید که مجبورم دست نویس کار کنم!!
اولی برای سوال اول خانم مهرنوش و دومی برای سوالات بعدی:
http://sfwdmotahari.persiangig.com/document/Libration.tif
http://sfwdmotahari.persiangig.com/document/1.tif

Mehrnoosh
04-19-2011, 01:04 AM
سلام و خداقوت:)
این روزها نزدیک امتحان مرحله ی دوم است و می دانم که بعدها این روزها برایتان خاطرات به یادماندنی ای خواهد بود...تلاش و زحمت و نشاط و امید و توکل و محبت و دعا و موفقیت و انرژی و شور و آینده و آرامش و ...کلماتی که این روزها زیاد می شنوید...ان شاالله بهترین درس ها را با این کلمات یاد بگیریم...:)
من با اجازه تان در جواب سوالات اخیر چیزی نوشته ام بسیار مختصر که در لینک های زیر است...چون تایپ فرمول و ...خیلی وقت گیر است باید ببخشید که مجبورم دست نویس کار کنم!!
اولی برای سوال اول خانم مهرنوش و دومی برای سوالات بعدی:
http://sfwdmotahari.persiangig.com/document/Libration.tif
http://sfwdmotahari.persiangig.com/document/1.tif
:thanks::thanks:

Mehrnoosh
04-19-2011, 11:05 PM
http://irupload.ir/images/tdbihr698xgv58ap69lj.jpg
این پاسخ نامه ی خودشه
عکس دوم قسمتی که نوشته delta a/delta t به توان n ، رو دلیلشو نمی فهمم!!!

چرا به توان n ؟؟؟؟؟؟؟؟؟؟؟؟؟؟؟

Fowad
04-25-2011, 03:02 PM
سلام مجدد!
اگر منظورتان این است که چرا فرم توانی برای قانون که خوب این را دلخواه انتخاب کرده و البته اولین انتخابی است که به ذهن می رسد چون مثلا مقدار کاهش در پایان دور دوم می شود مقدار کاهش دور اول ضرب در مقدار کاهش دور دوم و به همین ترتیب برای دورهای بعدی...حالا این فرض کرده که فرم کلی به صورت توانی از ان باشد و خواسته آن انی را بیابد که در شرطی که در همان برگه ها عرض کردم٬صدق کند.
:)

Mehrnoosh
04-25-2011, 11:40 PM
http://irupload.ir/images/ktv69djyr89spijkq574.jpg
:help:
:grin:

Astronomer
04-26-2011, 12:34 AM
خوب با توجه به اين كه با تغيير ميل خورشيد از مثبت 23.5 تا منفي همان عدد، سمت از 45 تا 135 درجه شمالي تغيير ميكند ميتوان مسئله را حل كرد.
البته توجه كنيد كه براي عرض هاي جنوبي هم ميتوان عددي بدست آورد.

Mehrnoosh
04-26-2011, 09:50 AM
سمت از 45 تا 135 درجه شمالي تغيير ميكند
.
چرا؟؟؟
.....

Ehsan
04-26-2011, 10:51 AM
چرا؟؟؟
.....

باید حول ِ 90 درجه متقارن باشه! (45-90 و 45+90)

Mehrnoosh
04-26-2011, 10:00 PM
اگر تغییرات میل خورشید در طول یک روز را در نظر بگیریم زمانی که خورشید در بیشترین ارتقاع خود قرار دارد با زمانی که خورشید روی نصف النهار قرار دارد اختلاف دارد
الف) حد اکثر این اختلاف در چه روزی هست ؟
ب) حداکثر مقدار این اختلاف رو بدست آورید.
:help:

پیام بهرام پور
04-27-2011, 10:05 PM
اگر تغییرات میل خورشید در طول یک روز را در نظر بگیریم زمانی که خورشید در بیشترین ارتقاع خود قرار دارد با زمانی که خورشید روی نصف النهار قرار دارد اختلاف دارد
الف) حد اکثر این اختلاف در چه روزی هست ؟
ب) حداکثر مقدار این اختلاف رو بدست آورید.
:help:

فایلی در رابطه با محاسبه ی میل خورشید برای شما ضمیمه ی این نامه می کنم که فکر کنم به پاسخ خودتان برسید
فایل از کتاب الفبای المپیاد نجوم و اخترفیزیک انتخاب شده است (جلد اول)

پیام بهرام پور
07-29-2011, 03:16 PM
چطور سوالات دوستان ته کشید؟
یه مدت فروغ خوبی داشتینا!
چرا المپیادی ها فقط موقع امتحان به یاد سوالاشون می افتن؟

solh
07-30-2011, 05:39 PM
چطور سوالات دوستان ته کشید؟
یه مدت فروغ خوبی داشتینا!
چرا المپیادی ها فقط موقع امتحان به یاد سوالاشون می افتن؟

سوال كه زياده
من راستش در پاسخ گويي به يكي از سوالات فصل گرانش هاليدي ناكام ماندم ،چون هر كاري كه ميكردم تعداد معادلات از تعداد مجهولات كمتر ميشد ، سوال رو مينويسم فقط تا اونجايي كه ممكنه جواب سوال را بتوانيم از مطالبي كه در هاليدي گفته شده استخراج كنيم بهتره . ممنون از دوستان .
...گاهي ميتوان حضور يك سياره نامرئي كه دور يك ستاره دور ميچرخد را از حركت ظاهري ان ستاره نتيجه گيري كرد . همچنان كه ستاره و سياره حول مركز جرم سيستم ستاره _سياره دوران ميكنند ستاره نسبت به ما دور و نزديك ميشود و حركتي كه ان را سرعت خط ديد مينامند ايجاد ميكند . اين حركت قابل مشاهده است .شكل 13_50نمودار سرعت خط ديد را بر حسب زمان براي هركول14نشانه ميدهد . جرم ستاره را حدود 0.90جرم خورشيد تخمين ميزنند .فرض كنيد فقط يك سياره به دور ستاره ميگردد و ديد ما در امتداد صفحه مدار است . الف)جرم سياره بر حسب جرم مشتري (لطفا خود جرم سياره را اگر ميشود بگوييد ) ب)شعاع مداري سياره را بر حسب شعاع مداري زمين تخمين بزنيد (لطفا خود شعاع مداري را به دست اوريد ) .
در مورد نمودار13_50:ماكسيمم سرعت 70 متر بر ثانيه . مينيمم سرعت منفي هفتاد متر بر ثانيه ، نمودار به شكل نمودار سينوسي بوده و دوره تناوب سرعت (از مثبت 70 تا مثبت 70 ديگر)1500ثانيه(در خود كتاب واحد گذاشته نشده) است .
لطفا جواب بدهيد . اگر به صورت توضيح باشد بهتر است . همانطور كه گفتم انگار تعداد مجهولات بيشتر از تعداد معادلات است .

celestial boy
07-30-2011, 05:57 PM
فرمول تبديل مختصات استوايي به افقي چيه.يك چيزي مشابه اون رو ديدم اما هرچي امتحانش كردم درست در نيامد.احتمالا يك چيزي ازش اشتباه بود.

Astronomer
07-30-2011, 06:07 PM
سوال كه زياده
من راستش در پاسخ گويي به يكي از سوالات فصل گرانش هاليدي ناكام ماندم ،چون هر كاري كه ميكردم تعداد معادلات از تعداد مجهولات كمتر ميشد ، سوال رو مينويسم فقط تا اونجايي كه ممكنه جواب سوال را بتوانيم از مطالبي كه در هاليدي گفته شده استخراج كنيم بهتره . ممنون از دوستان .
...گاهي ميتوان حضور يك سياره نامرئي كه دور يك ستاره دور ميچرخد را از حركت ظاهري ان ستاره نتيجه گيري كرد . همچنان كه ستاره و سياره حول مركز جرم سيستم ستاره _سياره دوران ميكنند ستاره نسبت به ما دور و نزديك ميشود و حركتي كه ان را سرعت خط ديد مينامند ايجاد ميكند . اين حركت قابل مشاهده است .شكل 13_50نمودار سرعت خط ديد را بر حسب زمان براي هركول14نشانه ميدهد . جرم ستاره را حدود 0.90جرم خورشيد تخمين ميزنند .فرض كنيد فقط يك سياره به دور ستاره ميگردد و ديد ما در امتداد صفحه مدار است . الف)جرم سياره بر حسب جرم مشتري (لطفا خود جرم سياره را اگر ميشود بگوييد ) ب)شعاع مداري سياره را بر حسب شعاع مداري زمين تخمين بزنيد (لطفا خود شعاع مداري را به دست اوريد ) .
در مورد نمودار13_50:ماكسيمم سرعت 70 متر بر ثانيه . مينيمم سرعت منفي هفتاد متر بر ثانيه ، نمودار به شكل نمودار سينوسي بوده و دوره تناوب سرعت (از مثبت 70 تا مثبت 70 ديگر)1500ثانيه(در خود كتاب واحد گذاشته نشده) است .
لطفا جواب بدهيد . اگر به صورت توضيح باشد بهتر است . همانطور كه گفتم انگار تعداد مجهولات بيشتر از تعداد معادلات است .
خوب راهنمايي براي اين مسئله منجر به حل مستقيم اونه! پس بهتره قبل از راهنمايي شما مجهولاتتون و معادلاتي كه در ذهن داريد را اينجا بيان كنيد. ببينم كدوم معادله رو جا انداختيد.

توجه: چون نمودار سرعت شعاعي كاملا سينوسي است مدار سياره دايره اي است.

solh
07-30-2011, 10:34 PM
خوب راهنمايي براي اين مسئله منجر به حل مستقيم اونه! پس بهتره قبل از راهنمايي شما مجهولاتتون و معادلاتي كه در ذهن داريد را اينجا بيان كنيد. ببينم كدوم معادله رو جا انداختيد.

توجه: چون نمودار سرعت شعاعي كاملا سينوسي است مدار سياره دايره اي است.
حالا كه دوباره نگاه ميكنم انگار يك نكته ي ساده رو يادم رفته . اگه درست بگم فاصله از مركز جرم ستاره رو با استفاده از دوره تناوب و ماكسيمم سرعت ستاره كه همون سرعت گردش ستاره است به دست مي ياد و بعد با استفاده از قانون كپلر جرم سياره و با تناسب جرم و فاصله از مركز جرم سيستم ميتوانيم فاصله ي سياره رو از مركز جرم به دست بياريم . اين راه حل درسته ؟

Astronomer
07-31-2011, 09:49 AM
حالا كه دوباره نگاه ميكنم انگار يك نكته ي ساده رو يادم رفته . اگه درست بگم فاصله از مركز جرم ستاره رو با استفاده از دوره تناوب و ماكسيمم سرعت ستاره كه همون سرعت گردش ستاره است به دست مي ياد و بعد با استفاده از قانون كپلر جرم سياره و با تناسب جرم و فاصله از مركز جرم سيستم ميتوانيم فاصله ي سياره رو از مركز جرم به دست بياريم . اين راه حل درسته ؟
دقيقا راه دذسته ولي بايد يك نكته رو فراموش نكنيد و اون اينكه اين سرعتس كه شما داريد براي ستاره هست نه سياره(درسته؟ فكر كنم سؤال رو اينطوري گفتيد) اونوقت بايد يادتون باشه كه سرعت ها هم متناسب با عكس نسبت جرم ها خواهند بود و بايد سرعت سياره رو استخراج كنيد اما دوره تناوب براي هردو يكيست و همينطور كپلر(البته ميتونيد از سرعت حركت دايره اي هم استفاده كنيد فكر كنم)

solh
07-31-2011, 11:42 AM
دقيقا راه دذسته ولي بايد يك نكته رو فراموش نكنيد و اون اينكه اين سرعتس كه شما داريد براي ستاره هست نه سياره(درسته؟ فكر كنم سؤال رو اينطوري گفتيد) اونوقت بايد يادتون باشه كه سرعت ها هم متناسب با عكس نسبت جرم ها خواهند بود و بايد سرعت سياره رو استخراج كنيد اما دوره تناوب براي هردو يكيست و همينطور كپلر(البته ميتونيد از سرعت حركت دايره اي هم استفاده كنيد فكر كنم)
من هم از رابطه ي سرعت و دوره در دايره استفاده كردم . ممنون .

celestial boy
08-01-2011, 06:00 PM
پس جواب سوال من چي شد.منتظرم.

Astronomer
08-01-2011, 06:38 PM
فرمول تبديل مختصات استوايي به افقي چيه.يك چيزي مشابه اون رو ديدم اما هرچي امتحانش كردم درست در نيامد.احتمالا يك چيزي ازش اشتباه بود.

پس جواب سوال من چي شد.منتظرم.

2052
اين از جواب سؤالتون! براي محاسبه ميل در فرمول اول از يك فرمول كسينوس ها در مثلث كروي pzx و براي زاويه ساعتي از يك فرمول چهرجزئي در همان مثلث با چهارجزء سمت، متمم عرض چغرافيايي، زاويه ساعتي و متمم ميل استفاده شده است.

براي تبديل زاويه ساعتي به بعد هم نياز به زمان نجومي است كه به نحوي بايد در صورت سؤال آمده باشد.

celestial boy
08-03-2011, 06:31 PM
ممنون.يه مقداري سخته اما سعي مي كنم بفهمم.
راستي از كتاب نجوم كروي كجاهاش رو لازمه بخونم و كجاهاش رو بهتره بخونم؟
بازم ممنون.

doostesetareha
08-03-2011, 08:41 PM
از کتاب نجوم کروی فصل های یک و دو و پنج و یازده - اگه اشتباه نکرده باشم - حتماً باید خونده بشه. یعنی طوریه که خیلی خیلی زیاد ب آدم کمک میکنه و اینارو بلد نباشی یعنی هیچی بلد نیستی !! بعد فصل های 3 و شاید 14 هم بخونین خوبه. یعنی اگه دیدین رو اینا تسلط کافی دارین اضافی بخونین بد که نیست هیچ خیلی هم خوبه !

Amin-Mehraji
08-03-2011, 09:32 PM
یک سؤال بسیار سخت . المپیاد دانشجویی نجوم هم داریم ؟؟!!

اگه داریم حتما باید دانشجوی همون رشته ( یعنی نجوم ) باشی تا بتونیم شرکت کنیم ؟

پیام بهرام پور
08-03-2011, 11:58 PM
یک سؤال بسیار سخت . المپیاد دانشجویی نجوم هم داریم ؟؟!!

اگه داریم حتما باید دانشجوی همون رشته ( یعنی نجوم ) باشی تا بتونیم شرکت کنیم ؟
سلام!
نه خیر نداریم!
تنها چیزی که در ایران شبیه به المپیاد نجوم هست (خیلی در سطح پایین تر) مسابقه سراسری نجوم می باشد که برای همه قابل شرکت کردن است!

celestial boy
08-04-2011, 10:44 AM
اين اثبات هايي كه در كتاب نجوم كروي نوشته رو حتما بايد ياد بگيرم يا فقط فرمول ها كافيه.چون اصلا از اثبات فرمولاش چيزي سر در نميارم.فكر كنم براي من زود باشه.
يه سوال ديگه.
چند تا المپياد نجوم ديگه تو ايران برگذار مي شه؟يه كم در مورد اونا توضيح بديد.ممنون

Astronomer
08-04-2011, 11:03 AM
اين اثبات هايي كه در كتاب نجوم كروي نوشته رو حتما بايد ياد بگيرم يا فقط فرمول ها كافيه.چون اصلا از اثبات فرمولاش چيزي سر در نميارم.فكر كنم براي من زود باشه.
يه سوال ديگه.
چند تا المپياد نجوم ديگه تو ايران برگذار مي شه؟يه كم در مورد اونا توضيح بديد.ممنون
اون اثبات هايي كه ميفرماييد براي فصل 1 بايد باشه!!! كه صد در صد نيازي به اون ها نيست.
از فصل 1 فقط با حفظ كردن چهار فرمول الف-ب-ج-د بگذريد و از اونها در فصل 2 استفاده كنيد.
اگر كتاب نجوم كروي براي شما سنگين است بنده كتاب هايي ساده تر چون اصول و عمل و كتاب جناب بهرامپور رو توصيه ميكنم.

المپياد نجوم كلا يدونست تو ايران! يه آزمون سراسري نجوم هم داريم كه همه ميتونن در اون شركت كنن..

celestial boy
08-04-2011, 12:45 PM
ممنون از پاسختون.
كتاب اصول و عمل رو كه فكر نكنم بتوانم پيداش كنم.كتاب آقاي بهرام پور هم به زودي به دستم مي رسه.اما اين كتاب به همون اندازه نجوم كروي مفيد و كافي هست؟
منظورم المپياد هاي ديگه اي مثل المپيادي كه از طرف دفتر آيت الله سيستاني برگذار مي شه.

sooshans
08-04-2011, 04:31 PM
ممنون از پاسختون.
كتاب اصول و عمل رو كه فكر نكنم بتوانم پيداش كنم.كتاب آقاي بهرام پور هم به زودي به دستم مي رسه.اما اين كتاب به همون اندازه نجوم كروي مفيد و كافي هست؟
منظورم المپياد هاي ديگه اي مثل المپيادي كه از طرف دفتر آيت الله سيستاني برگذار مي شه.
نه طبیعتا اینطور نیس ولی برا شروع بشدت خوبه:thumbsup:ولی اصول و عمل تقریبا در سطح نجوم کروی است در بعضی موارد پایین تر;)
اون المپیاد نیس مسابقه سراسری نجومه ما یه دونه المپ بیشتر نداریم:pathead:

solh
08-04-2011, 05:02 PM
من هم تازه شرع كردم نجوم كروي بخونم . تا اندازه اي روي فصل يك و دو مسلط شدم . البته زياد هم كه نه چون هنوز دارم روي فصل دو كار ميكنم . سوال گير نمي ياد . من هم كه تا سوال حل نكنم انگار نه انگار . بگم كه در اين مدت كم كه شروع كردم بسيار زياد به اين كتاب (نجوم كروي)علاقه مند شدم . كتاب شيرين و جذابي است و تا حالا به بسياري از سوالات من پاسخ داده . تصميم دارم تا اونجايي كه ميتونم به طور كامل بخونمش (هر چند كه بعضي فصل ها رو به خاطر كمبود وقت بايد سطحي بخونم ) . اثبات كنيد هاشم سخت نيست به شرطه اينكه منظور سوال رو بفهمي كه خودش يه گير اساسيه ،من در بعضي سوال ها هر چقدر تلاش ميكنم نميفهمم دقيقا خواست مسئله چيه .

doostesetareha
08-04-2011, 06:28 PM
برا نمونه سوال بهترین نمونه سوالای سال های قبله. چه مرحله اول و چه مرحله دوم... در ضمن اگه یه کم دنبال سایتای المپیادیای سالای قبل باشین - اسم نمیارم ک تبلیغ میشه !! - یه سری آزمونارو تو وبلاگاشون دارن که اونام خوبن.
همین دیگه فعلاً !!

Astronomer
08-04-2011, 06:34 PM
من هم تازه شرع كردم نجوم كروي بخونم . تا اندازه اي روي فصل يك و دو مسلط شدم . البته زياد هم كه نه چون هنوز دارم روي فصل دو كار ميكنم . سوال گير نمي ياد . من هم كه تا سوال حل نكنم انگار نه انگار . بگم كه در اين مدت كم كه شروع كردم بسيار زياد به اين كتاب (نجوم كروي)علاقه مند شدم . كتاب شيرين و جذابي است و تا حالا به بسياري از سوالات من پاسخ داده . تصميم دارم تا اونجايي كه ميتونم به طور كامل بخونمش (هر چند كه بعضي فصل ها رو به خاطر كمبود وقت بايد سطحي بخونم ) . اثبات كنيد هاشم سخت نيست به شرطه اينكه منظور سوال رو بفهمي كه خودش يه گير اساسيه ،من در بعضي سوال ها هر چقدر تلاش ميكنم نميفهمم دقيقا خواست مسئله چيه .
كتاب نجوم كروي نوشته اسمارت بسيار كتاب خوب و زيبايي است ولي هرگز و هرگز براي المپياد نيازي نيست كه اون رو به طور كامل بخونيد... تعدادي از فصل ها رو در چند پست قبل يكي از كاربران گفته!
همينطور در صفحات قبل لينكي از مجموعه سؤالات نجومي موجود در المپياد رو قرار داديم كه به همت دوست خوب و كاربر فروم "علي ايزدي راد" جمع آوري شده!

من به شدت به اين بخش نجوم علاقه دارم چون سؤال هاي ايده اي بسيار خوبي در اون موجود هست. براي پيشرفت و موفقيت در اين بخش بنده رفتن به سر كلاس يك استاد رو پيشنهاد ميكنم. نجوم كروي رو بايد با استاد ياد گرفت.(ياد استادمون بخير! چقدر اين قسمت رو باحال درس ميداد!!!)

solh
08-04-2011, 11:06 PM
كتاب نجوم كروي نوشته اسمارت بسيار كتاب خوب و زيبايي است ولي هرگز و هرگز براي المپياد نيازي نيست كه اون رو به طور كامل بخونيد... تعدادي از فصل ها رو در چند پست قبل يكي از كاربران گفته!
همينطور در صفحات قبل لينكي از مجموعه سؤالات نجومي موجود در المپياد رو قرار داديم كه به همت دوست خوب و كاربر فروم "علي ايزدي راد" جمع آوري شده!

من به شدت به اين بخش نجوم علاقه دارم چون سؤال هاي ايده اي بسيار خوبي در اون موجود هست. براي پيشرفت و موفقيت در اين بخش بنده رفتن به سر كلاس يك استاد رو پيشنهاد ميكنم. نجوم كروي رو بايد با استاد ياد گرفت.(ياد استادمون بخير! چقدر اين قسمت رو باحال درس ميداد!!!)
خوب در مورد استاد !!ما شهرستاني ها چيكار بايد بكنيم ؟استاد فيزيك نداريم (جدي ميگم)اونوقت استاد نجوم ،اونم نجوم كروي رو از كجا گير بياريم . در ضمن من كه كتاب هاي زيادي ميخونم كه ربطي به المپياد نداره ،اون فصول از نجوم كروي هم روش.(به عنوان تفريح و وقت ازاد مناسب است!!!نيست؟)

Astronomer
08-05-2011, 09:17 AM
خوب در مورد استاد !!ما شهرستاني ها چيكار بايد بكنيم ؟استاد فيزيك نداريم (جدي ميگم)اونوقت استاد نجوم ،اونم نجوم كروي رو از كجا گير بياريم . در ضمن من كه كتاب هاي زيادي ميخونم كه ربطي به المپياد نداره ،اون فصول از نجوم كروي هم روش.(به عنوان تفريح و وقت ازاد مناسب است!!!نيست؟)
بنده گفتم نيازي نيست اما اگر شما وقت آزاد داريد كه بخونيد مشكلي نيست و البته بگم كه كتاب سنگيني هست و به هركس توصيه نميكنم...

شما خودتون تا اونجا كه ميتونيد بخونيد و پيش بريد و مسئله حل كنيد! هرجا كه به مشكل برخورديد ميتونيد از كلاس هاي رفع اشكال بهره بگيريد. مثلا همين الآن تعداد كساني كه در اين فروم المپيادي هستند و نياز به كلاس هاي رفع اشكال دارند در حد يه كلاس كوچك است. با هم در تاپيك جامع المپياد نجوم آشنا بشيد در حالي كه هركدام براي يك شهرستانيد و از آوا بخواهيد كه براتون كلاس رفع اشكال برگزار كنه... اونوقت فقز با ماهي يك يا دو روز در تهران بودن همون مقدار كه بنده استاد تجربه كردم تجربه ميكنيد(استاد ما ماهي دوروز به شهرمون ميومد!) اما بايد براي لحظه لحظه اون كلاس برنامه داشته باشيد و استفاده كنيد.

من نمي دونم چرا اين بچه هاي شهرستاني اينقده ميترسن از بي مدرسي...بخونيد و اشكالاتتون رو مطرح كنيد. جواب ميگيريد!

celestial boy
08-08-2011, 08:21 PM
يه سوال كلي در باره محاسبات نجوم كروي برام پيش اومده.
بيشتر اطلاعاتي كه در باره مثلا بعد و ميل يك ستاره در مسئله هست درجه كامل نيستند و دقيقه و ثانيه دارند.براي محاسبه اونا تو ماشين حساب هم كه نمي شه بدون تبديل به درجه حسابشون كرد.وقتي هم كه جواب رو بدست مياريم بايد همون تبديل رو انجام بديم.تو اين تبديل ها يك مشكل اساسي واسم پيش اومده.بعضي جا ها هم كه دقيقه ها با هم فرق دارن.(البته فك كنم.)لطفا راهشو برام توضيح بدين
ممنون...

solh
08-08-2011, 10:35 PM
يه سوال كلي در باره محاسبات نجوم كروي برام پيش اومده.
بيشتر اطلاعاتي كه در باره مثلا بعد و ميل يك ستاره در مسئله هست درجه كامل نيستند و دقيقه و ثانيه دارند.براي محاسبه اونا تو ماشين حساب هم كه نمي شه بدون تبديل به درجه حسابشون كرد.وقتي هم كه جواب رو بدست مياريم بايد همون تبديل رو انجام بديم.تو اين تبديل ها يك مشكل اساسي واسم پيش اومده.بعضي جا ها هم كه دقيقه ها با هم فرق دارن.(البته فك كنم.)لطفا راهشو برام توضيح بدين
ممنون...

در اخر فصل يك نجوم كروي يه كم توضيح داده اگه دوباره نگاه كني متوجه ميشي.
خودت هم ميتوني محاسبه كني ، مثلا 24 ساعت طول ميكشد تا خورشيد 360درجه رو طي كنه كه از اون نتيجه ميشه هر ساعت برابر 15 درجه هست و ...
در مثلثات هر درجه برابر 60دقيقه و هر دقيقه برابر 60 ثانيه (قوسي )هست . با استفاده از همين داده ها ثانيه و دقيقه روي اسمان هم به دست مياد كه همونطور كه در نجوم كروي ديده ميشه در اسمان هر دقيقه برابر 15دقيقه قوسي (يك چهارم يك درجه)و هر ثانيه برابر 15ثانيه قوسي (يك چهارم دقيقه قوسي)هست .

پیمان اکبرنیا
08-08-2011, 11:15 PM
يه سوال كلي در باره محاسبات نجوم كروي برام پيش اومده.
بيشتر اطلاعاتي كه در باره مثلا بعد و ميل يك ستاره در مسئله هست درجه كامل نيستند و دقيقه و ثانيه دارند.براي محاسبه اونا تو ماشين حساب هم كه نمي شه بدون تبديل به درجه حسابشون كرد.وقتي هم كه جواب رو بدست مياريم بايد همون تبديل رو انجام بديم.تو اين تبديل ها يك مشكل اساسي واسم پيش اومده.بعضي جا ها هم كه دقيقه ها با هم فرق دارن.(البته فك كنم.)لطفا راهشو برام توضيح بدين
ممنون...

فرض کنید میخواهیم 15 درجه و 36 دقیقه و 47 ثانیه را تبدیل به درجه و کسری از آن کنیم. مراحل به صورت زیر است:
1- 47 ثانیه را تقسیم بر 60 میکنیم که میشه 0.78
2- این مقدار را به دقیقه ها اضافه میکنیم که می شود 36.78
3- حال 36.78 را تقسیم بر 60 میکنیم که میشود 0.613
4- این مقدار را به درجه ها اضافه میکنیم میشود 15.613 درجه

حال فرض کنید می خواهیم 35.3674 درجه را به دقیقه و ثانیه تبدیل کنیم
1- 0.3674 را ضرب در 60 می کنیم که می شود 22.044
2- حال کسر 0.044 را در 60 ضرب میکنیم که می شود 2.64
3- پس زاویه می شود 35 درجه و 22 دقیقه و 3 ثانیه

امیدوارم تونسته باشم به خوبی توضیح دهم

celestial boy
08-09-2011, 11:04 AM
عالي بود.خيلي ممنون.
فكر كنم خودم برم دنبال جواب سوالام بهتر باشه.اين جوري تنبل مي شم و هر چي سوال دارمو تو آوا مي نويسم.فقط كافيه يكم به كله ام فشار بيارم.ديگه شرمنده شما هم نمي شم.
خيلي ممنون...

solh
08-29-2011, 10:53 PM
يه سوال داشتم : سرعتي كه از معادله دوپلر به دست مياد سرعت نسبت به زمينه ديگه ؟ تگه بخوايم اين سرعت رو نسبت به خورشيد و مركز كهكشان تبديل كنيم بايد چيكار كنيم ؟ تصحيحات مربوط به سرعت شعاعي نالشي از سرعت زمين بايد روي سرعت شعاعي ناشي از تغيير طول موج اعمال بشه ؟ اگه ميشه چجوري؟

پیمان اکبرنیا
08-30-2011, 12:05 AM
يه سوال داشتم : سرعتي كه از معادله دوپلر به دست مياد سرعت نسبت به زمينه ديگه ؟ تگه بخوايم اين سرعت رو نسبت به خورشيد و مركز كهكشان تبديل كنيم بايد چيكار كنيم ؟ تصحيحات مربوط به سرعت شعاعي نالشي از سرعت زمين بايد روي سرعت شعاعي ناشي از تغيير طول موج اعمال بشه ؟ اگه ميشه چجوري؟

اگر بخواهید سرعت را نسبت به خورشید حساب کنید به یک سری روابط نجوم کروی نیاز دارید. باید مختصات جسم در آسمان را بدانید و جای زمین در مدارش را هم بدانید و بعد زاویه راستای سرعت زمین با راستای خط واصل زمین ستاره را پیدا کنی. یک مقدار محاسبه داره که نمیشه به راحتی اینجا نوشت. بعد که سرعت نسبی دو جسم در راستای خط واصل به دست اومد، اون میشه مبنا برای محاسبه جابه جایی دوپلر!

solh
08-30-2011, 06:01 PM
محاسبات مربوط رو از كجا ميتونم گير بيارم ؟

پیمان اکبرنیا
08-30-2011, 08:34 PM
محاسبات مربوط رو از كجا ميتونم گير بيارم ؟

راستش من الان منبعی را به یاد ندارم که دقیقا این محاسبه را داشته باشه. شاید سایر دوستان المپیادی منبعی را بشناسند.

selenia
11-15-2011, 12:52 PM
سوالی در رابطه با مسائل اختر فیزیکی داشتم و ممنون میشم اگه منو در حل این مسئله راهنمایی کنید.


منحنی های سرعت یک دوتایی طیف سنجی دو خطی به صورت سینوسی با دامنه های20 کیلومتر بر ثانیه و60 کیلومتر بر ثانیه و دوره تناوب 1.5سال مشاهده میشود

خروج از مرکز مداری چقدر است؟*

کدام ستاره سنگین تر است و نسبت جرم های ستاره ای چقدر است؟*

.اگر میل مداری 90درجه باشد نیم محور بلند نسبی را بر حسب واحد نجومی بیابید

پیمان اکبرنیا
11-27-2011, 07:45 PM
سوالی در رابطه با مسائل اختر فیزیکی داشتم و ممنون میشم اگه منو در حل این مسئله راهنمایی کنید.


منحنی های سرعت یک دوتایی طیف سنجی دو خطی به صورت سینوسی با دامنه های20 کیلومتر بر ثانیه و60 کیلومتر بر ثانیه و دوره تناوب 1.5سال مشاهده میشود

خروج از مرکز مداری چقدر است؟*

کدام ستاره سنگین تر است و نسبت جرم های ستاره ای چقدر است؟*

.اگر میل مداری 90درجه باشد نیم محور بلند نسبی را بر حسب واحد نجومی بیابید

سلام

اگر منحنی شما دو خطی(مربوط به 2 ستاره) باشد و میل مداری هم 90 درجه باشد و نمودار هم کاملا سینوسی (متقارن) باشد، یعنی مدارها دایره ای است و خروج از مرکز صفر خواهد بود. اگر مدارها بیضی بود، باید اطلاعات بیشتری از منحنی های سرعت داشتیم تا میتونستیم خروج از مرکز را حساب کنیم.

2934

حالا با فرض دایره ای بودن مدار، میدانیم که نسبت سرعت بیشینه 1 به سرعت بیشینه 2، برابر است با معکوس نسبت جرمهای آنها. یعنی در این مثال، جرم اونی که سرعتش 20 کیلومتر بر ثانیه است، 3 برابر جرم اونی هست که سرعتش 60 هست.

برای پیدا کردن اندازه نیم محور بزرگ هم، باید از شکل نیوتونی رابطه کپلر استفاده کرد. یعنی

2932

البته اینجا نه مجموع جرمها را داریم و نه a را بنابر این از رابطه زیر استفاده میکنیم(که اثباتش نمیکنم ولی نسبتا ساده است و از رابطه بالا و جای گذاری روابط سرعت مداری به دست می آید)

2933

در رابطه بالا سرعت مداری یکی از اجرام و دوره تناوب را قرار میدهیم، i هم به گفته مساله 90 درجه است و سینوس آن 1 است. حال رابطه ای بین m1 و m2 به دست می آید. نسبت m1 به m2 را هم که به دست آورده بودیم پس با 2 معادله و 2 مجهول، جرمها به دست می آید.

حالا که جرمها به دست آمد در رابطه کپلر میگذاریم و نیم قطر یا همان a را به دست می آوریم.

selenia
12-01-2011, 06:52 PM
یه سوال در رابطه دوم باید m2رو بر حسب m1حساب کنیم و تو مخرج بذاریم؟

پیمان اکبرنیا
12-01-2011, 11:15 PM
یه سوال در رابطه دوم باید m2رو بر حسب m1حساب کنیم و تو مخرج بذاریم؟

بله یا برعکس m1 را بر حسب m2 جایگذاری کنید و بعد با کمی محاسبه و صحیح و خطا(چون معادله درجه 3 است)، مقدار m2 را پیدا کنید.

galaxy 1996
12-21-2011, 06:48 PM
این سه تا سوال رو هر چقدر میشه کامل برام جواب بدید.
یه دنیا ممنون.
307230733074

Astronomy
12-22-2011, 10:37 AM
این سه تا سوال رو هر چقدر میشه کامل برام جواب بدید.
یه دنیا ممنون.
307230733074
سلام
فکر نمی کنم که درست باشه اینجوری کل سوالای مسابقات دانش آموزی شریف رو بپرسین
پس تلاش خودتون چی میشه؟

Mostafa
12-22-2011, 11:17 AM
انشاالله بعد از پایان مسابقه دانش آموزی شریف ، پاسخ سوالات اینجا منتشر خواهد شد

galaxy 1996
12-22-2011, 07:43 PM
دوست عزیز!!!
من توی مسابقات شرکت نکرده ام!!!
برای همین اینجا مطرح کردم.
نمیتونم صبر کنم.
میخوام بدونم حلشون چطوریه.
بازم ممنون میشم پاسخ بدید.

Astronomy
12-22-2011, 08:39 PM
دوست عزیز!!!
من توی مسابقات شرکت نکرده ام!!!
برای همین اینجا مطرح کردم.
نمیتونم صبر کنم.
میخوام بدونم حلشون چطوریه.
بازم ممنون میشم پاسخ بدید.
اما شاید کسانی در اینجا باشند که در این مسابقه شرکت کرده اند (از جمله خودم) و جواب سوالات رو ببینند
انشالله سه شنبه که مهلت پاسخگویی تمام شد با هم سوالات رو مرور می کنیم

galaxy 1996
12-23-2011, 09:51 AM
پس اگه میشه همون سه شنبه بزارید من داره اعصابم میریزه به هم!!!
میخوام جواب ها رو بدونم.
باز هم ممنون.
یا علی

Anita
12-23-2011, 02:57 PM
این سه تا سوال رو هر چقدر میشه کامل برام جواب بدید.
یه دنیا ممنون.
307230733074


*توی صورت یکی از این سوالا گفته شده"این نقشه در مختصات بعد -میلی رسم شده به طوری که محور عمودی
مانند طول جغرافیایی و دامنه تغییرات ان از صفر تا 360 است" ولی مگه نباید میگفت از 0 تا 180؟؟
صورت سوال اشکال نداره؟

پیمان اکبرنیا
12-23-2011, 03:28 PM
*توی صورت یکی از این سوالا گفته شده"این نقشه در مختصات بعد -میلی رسم شده به طوری که محور عمودی
مانند طول جغرافیایی و دامنه تغییرات ان از صفر تا 360 است" ولی مگه نباید میگفت از 0 تا 180؟؟
صورت سوال اشکال نداره؟

فکر کنم صورت سوال درست باشه. منظورش اینه که از بالا تا پایین محور عمودی، 360 درجه است(یک دور کامل تغییرات بعد).

Anita
12-23-2011, 03:59 PM
فکر کنم صورت سوال درست باشه. منظورش اینه که از بالا تا پایین محور عمودی، 360 درجه است(یک دور کامل تغییرات بعد).

ممنون ولی مگه تغییرات بعد 0تا 180 نیست؟؟

Astronomy
12-23-2011, 07:42 PM
ممنون ولی مگه تغییرات بعد 0تا 180 نیست؟؟
نخیر بعد هم مانند دیگران 360 درجه دارد از 0 تا 360 که یک دور کامل می باشد

mobi
12-23-2011, 10:30 PM
ممنون ولی مگه تغییزات بعد 0 تا 180 نیست؟؟
------------------------------------------------------------
همون طور که جناب استرونومی گفتن بعد از 0 تا 360!!!و همچنین بعد رو با ساعت هم اعلام میکنن و هر ساعت معادل 15 درجه!!اگه 24 رو در 15 ضرب کنید میشود 360!!

solh
12-31-2011, 10:11 PM
یه سوال : تغیرات چگالی در ستاره ها در رابطه با شعاع روند خاصی رو داره ؟ اصولا میتونیم تابع چگالی رو با اطلاعات بدست اومده از رصد بنویسیم .

پیمان اکبرنیا
12-31-2011, 10:56 PM
یه سوال : تغیرات چگالی در ستاره ها در رابطه با شعاع روند خاصی رو داره ؟ اصولا میتونیم تابع چگالی رو با اطلاعات بدست اومده از رصد بنویسیم .

بله معادلاتی هستند که به کمک آنها، میشه تغییرات چگالی و سایر خصوصیات را از سطح تا مرکز ستاره محاسبه کرد. معادلات دیفرانسیلی و انتگرالی هستند و در کتابهای اخترفیزیک ستارگان پیدا می شوند. فکر کنم در اون کتاب 3 جلدی اخترفیزیک ستاره ای موجود باشند. یا در کتابهای اخترفیزیک معروفی مثل شد an introduction to modern astrophysics

solh
01-13-2012, 06:17 PM
برای ثبت نقطه ای ستاره ای مشخص در مختصات ، چقدر نور دهی لازمه ؟ میتونیم با مشاهده طول رد ستاره مشخصی در عکس، زمان نوردهی رو تخمین بزنیم ؟

Astronomy
01-13-2012, 06:23 PM
برای ثبت نقطه ای:
t=1000/(f*X)
که f فاصله کانونی و X میل مرکز تصویر هست و البته t هم زمان نوردهی هست
در مورد سوال دوم هم با دونستن مقدار زاویه کمان رد ستاره میشه زمان نوردهی رو بدست اورد

solh
01-14-2012, 10:05 PM
برای ثبت نقطه ای:
t=1000/(f*X)
که f فاصله کانونی و X میل مرکز تصویر هست و البته t هم زمان نوردهی هست
در مورد سوال دوم هم با دونستن مقدار زاویه کمان رد ستاره میشه زمان نوردهی رو بدست اورد
واحد ها چین ؟

Astronomy
01-14-2012, 11:14 PM
خیلی خیلی عذر می خوام
یه اشتباه کوجیک باعث شد همه چی بهم بریزه
فرمول درست اینه

t=1000/(f*cosX)

یک کوسینوس رو جا انداخته بودم
X میل مرکز عکس (درجه)
f فاصله کانونی لنز (میلیمتر)
t مدت زمان نوردهی برای ثبت نقطه ای (ثانیه)

solh
01-20-2012, 02:54 PM
دو جسم به جرم های m1وm2در فاصله ی d از هم قرار دارند و ساکنند ، اگر دو جسم بر اثر جاذبه ی گرانشی به هم بر خورد کنند ، چقدر طول میکشد تا با هم برخورد کنند ؟ (فاصله اجسام بسیار بزرگ تر از شعاع اجسام )

راه حلش رو هم بدین ممنون میشم .

shariatzadeh
01-20-2012, 03:10 PM
این سوال یک راه حل تحلیلی داره که توش به انتگرال نیاز پیدا میشه و یک راه حل خیلی ساده
راه حل ساده :بهتره که از جرم کاهیده استفاده کنید به گونه ای که جسمی به جرم mu=(m1*m2)/(m1+m2 در حال سقوط به روی جسمی با جرمM=m1+m2 هست
حالا فرض کنید مسیر جسم اول(mu) به دور جسم دوم(M) یک بیضی بسیار کشیده است که به خط تبدیل شده . حالا میتونیم بگیم نیم قطر اطول این مدار d/2 هست و از قانون سوم کپلر می تونید دوره تناوب این مدار (T) رو به دست بارید توجه کنید که در مخرج قانون سوم باید M=m1+m2 قرار بدید . دقت کنید جواب نصف دوره تناوب این مدار است یعنی T/2
راه حل دوم : از معادله مستقل از زمان (v dv=a dr) انتگرال بگیرید از d تا یک r دلخواه و اونطرف از صفر تا v دلخواه . حالا به جای v قرار بدید dr/dt بعد دوباره انتگرال بگیرید تا r بر حسب t بدست بیاد . فقط یه نکته ای توی این انتگرال باید از از جایگذاری مثلثاتی استفاده کنید و در تهایت به انتگرال 2^(sin x) می رسید .

solh
01-20-2012, 03:39 PM
این سوال یک راه حل تحلیلی داره که توش به انتگرال نیاز پیدا میشه و یک راه حل خیلی ساده
راه حل ساده :بهتره که از جرم کاهیده استفاده کنید به گونه ای که جسمی به جرم mu=(m1*m2)/(m1+m2 در حال سقوط به روی جسمی با جرمm=m1+m2 هست
حالا فرض کنید مسیر جسم اول(mu) به دور جسم دوم(m) یک بیضی بسیار کشیده است که به خط تبدیل شده . حالا میتونیم بگیم نیم قطر اطول این مدار d/2 هست و از قانون سوم کپلر می تونید دوره تناوب این مدار (t) رو به دست بارید توجه کنید که در مخرج قانون سوم باید m=m1+m2 قرار بدید . دقت کنید جواب نصف دوره تناوب این مدار است یعنی t/2
راه حل دوم : از معادله مستقل از زمان (v dv=a dr) انتگرال بگیرید از d تا یک r دلخواه و اونطرف از صفر تا v دلخواه . حالا به جای v قرار بدید dr/dt بعد دوباره انتگرال بگیرید تا r بر حسب t بدست بیاد . فقط یه نکته ای توی این انتگرال باید از از جایگذاری مثلثاتی استفاده کنید و در تهایت به انتگرال 2^(sin x) می رسید .
توی ماریون اینجور خواسته : دو ذره تحت تاثیر نیروی گرانش متقابلشان حرکت و مسیرهای دایره ای با دوره تناوب tرا به دور یکدیگر طی میکنند . اگر ناگهان در مدارهایشان متوقف شوند ، و امکان یابند که با جاذبه ی گرانشی به سوی یکدیگر کشیده شوند ، نشان دهید پس از مدت tتقسیم بر چهار رادیکال دو ، به یکدیگر برخورد خواهند کرد .

مگه در حالت اولیه قطر اطول در معادله کپلر دو برابر شعاع نمیشه ؟ اگه اینطور باشه با چیزی که شما میگین جور نمیشه .

shariatzadeh
01-20-2012, 08:48 PM
در حالت اول که در مدار دایروی با شعاع d دارن دور هم میگردن دوره تناوبشون میشه t
حالا توی یه مدار قرار می گیره که نیم قطر اطولشd/2 هست پس دوره تناوبش میشه tتقسیم بر 2 رادیکال 2 اما ما نصف این مقدار رو می خواهیم که میشه t تقسیم بر 4 رادیکال 2

solh
01-26-2012, 02:13 PM
اصلا اینجا کسی هست که المپیاد بخونه ؟ کجایید؟
توی لینک زیر درباره دستگاه مختصات ECEF و عناصر مدار صحبت کردم و همچنین درباره دستگاه perifocal هم توضیح بیشتری دادم .
خوندن این جزوه رو به خصوص به بچه هایی که به صورت جدی المپیاد می خونن توصیه می کنم
لطفا مسائل آخر جزوه رو کامل حل کنید ، اینجوری مطمئن میشید که یاد گرفتید یا نه . این قسمت برای کسایی که توی کار با ماشین حساب مشکل دارن خیلی تلخه ،لطفا دقت کنید ...
البته این لینک رو توی قسمت مکانیک سماوی هم گذاشتم ، ولی اینجا مخصوصه.

عناصر مدار.pdf (http://s1.picofile.com/file/7263425806/anasor_madar.pdf.html)


بله هستن !!

در ضمن حالا که نوشتم ، لازمه که تشکر هم بکنم ، واقعا ممنون .

راستی یه مشکلی که من خودم دارم اینه که توی مسائلی که مکانیک سماوی با کروی تلفیق میشه ، یه جورایی گیج میزنم!! انگار مطلب مشخصی وجود داره که باید کار بشه ، آیا چیز خاصی باید خونده بشه ؟ مثلا از داده هایی که توی آسمون در مورد حرکت بدست میاد (در رابطه با تغییرات مختصات )، بخایم ویژگی های حرکتی ستاره ، یا دوتایی رو بیرون بکشیم .

یه سوالی که قبلا هم پرسیدم : چجوری تصحیح مربوط به حرکت زمین رو برای انتقال دوپلری و تبدیل سرعت نسبی ، نسبت به زمین ، به سرعت نسبت به خورشید انجام میدیم ؟

shariatzadeh
01-26-2012, 07:33 PM
بسیار خوب . solh یه سوال خوب مطرح کرد ، ترجیح میدم روی سوال زیر که تحول یافته سوال بالا است همه فکر کنن ، تا جوابش رو بدم .

ستاره ای با بعد و میل α و δ را در نظر بگیرید . شخصی از روی زمین قرمزگرایی این ستاره را z اندازه می گیرد . اگر مدار زمین را بیضی با خروج از مرکز e و طول حضیض ω فرض کنیم و هم اکنون آنومالی واقعی زمین درمدارش θ باشد .مقدار واقعی قرمز گرایی ستاره چقدر است ؟
;)

solh
01-27-2012, 08:17 PM
تا حالا من به اینجا رسیدم :

با استفاده از آنومالی ، معادله کپلر و معادله مدار ، میتونیم سرعت زمین رو در مدار به دست بیاریم .

حالا زاویه ی بین امتداد سرعت زمین و خط واصل ستاره رو از توی کره سماوی پیدا میکنی ، که فکر کنم بشه زاویه ای که دایرت البروج با استوای سماوی میسازه ، بعلاوه ی زاویه ای که دایره ای عظیمه ی گذشته از ستاره با استوای سماوی میسازه .

حالا مختصه ی مربوط به سرعت زمین رو که روی جهت ستاره میفته رو پیدا میکنیم ( خط واصل ستاره محور y ها و مختصه ی منظور سرعت در سینوس زاویه بین هست . )

مقدار قرمز گرایی ناشی از این سرعت رو پیدا کرده از مقدار به دست آمده کم میکنیم .

(امیدوارم هزیان نگفته باشم !!)

ما منتظر پاسخیم !!

erfan bayat
01-27-2012, 10:06 PM
تا حالا من به اینجا رسیدم :

با استفاده از آنومالی ، معادله کپلر و معادله مدار ، میتونیم سرعت زمین رو در مدار به دست بیاریم .

حالا زاویه ی بین امتداد سرعت زمین و خط واصل ستاره رو از توی کره سماوی پیدا میکنی ، که فکر کنم بشه زاویه ای که دایرت البروج با استوای سماوی میسازه ، بعلاوه ی زاویه ای که دایره ای عظیمه ی گذشته از ستاره با استوای سماوی میسازه .

حالا مختصه ی مربوط به سرعت زمین رو که روی جهت ستاره میفته رو پیدا میکنیم ( خط واصل ستاره محور y ها و مختصه ی منظور سرعت در سینوس زاویه بین هست . )

مقدار قرمز گرایی ناشی از این سرعت رو پیدا کرده از مقدار به دست آمده کم میکنیم .

(امیدوارم هزیان نگفته باشم !!)

ما منتظر پاسخیم !!


سوال ۲۳ مجموعه سوالات نیما چرتاب رو ببین واسه راهنمائی خوبه
اشتباه نکنم به یه جاهائی‌ رسیدی ولی‌ پیشنهاد می‌کنم بردار سرعت ستاره و زمین رو بنویسی‌ که کارت راحت تر باشه
توی اینجور مسائل این کار راحتره

shariatzadeh
01-28-2012, 05:23 PM
خوبه یعنی بد نیست . یه نفر به فکرما هست
بردار کاربردهای زیادی در نجوم داره این سوال رو هم میتونیم با بردار حل کنیم .

1-برای استفاده از بردار باید یک دستگاه مختصات مناسب داشته باشیم . محور x راروی نقطه حضیض مدار زمین و محور z را روی قطبشمال دایره البروج قرار می دیم پس صفحه x-y صفحه دایرهالبروج است که مدار زمین رو شامل میشه .

2- با استفاده از بعد و میل داده شده طول و عرض دایره البروجی را بدست می آوریم .


3- بردار یکه مکان ستاره را می نویسیم


4- بردار سرعت زمین در راستای ستاره که سرعت اضافی است را با ضرب داخلی بردار سرعت زمین در بردار یکه مکان ستاره بدستمی آوریم. بردار سرعت زمین را در جزوه عناصر مدار به دست آورده ام .


5- قرمز گرایی اضافی را که ناشی از مولفه سرعت زمین در راستای ستاره است را تصحیح می کنیم.
3372

solh
01-28-2012, 06:20 PM
خوبه یعنی بد نیست . یه نفر به فکرما هست
بردار کاربردهای زیادی در نجوم داره این سوال رو هم میتونیم با بردار حل کنیم .

1-برای استفاده از بردار باید یک دستگاه مختصات مناسب داشته باشیم . محور x راروی نقطه حضیض مدار زمین و محور z را روی قطبشمال دایره البروج قرار می دیم پس صفحه x-y صفحه دایرهالبروج است که مدار زمین رو شامل میشه .

2- با استفاده از بعد و میل داده شده طول و عرض دایره البروجی را بدست می آوریم .


3- بردار یکه مکان ستاره را می نویسیم


4- بردار سرعت زمین در راستای ستاره که سرعت اضافی است را با ضرب داخلی بردار سرعت زمین در بردار یکه مکان ستاره بدستمی آوریم. بردار سرعت زمین را در جزوه عناصر مدار به دست آورده ام .


5- قرمز گرایی اضافی را که ناشی از مولفه سرعت زمین در راستای ستاره است را تصحیح می کنیم.
3372
با تشکر از جواب

چنتا سوال در مورد کلیت مسئله : آیا زاویه ی بین بردارها ی سرعت ، زاویه ی بین دایرت البروج و دایره ی عظیمه ی گذشته از ستاره است ؟

2) مقدار سرعت زمین رو چجوری بدست میاریم ؟ در واقع اینطور که من حل میکنم یه احتیاج برای حل معادله کپلر هست که زاویه رو پیدا کنیم . و من نمیتونم این معادله رو حل کنم ، روش دیگه ای وجود داره ؟

کلا حل معادله کپلر برای زاویه چجوریه؟

shariatzadeh
01-28-2012, 06:51 PM
سوال اول : خیر ،بردار سرعت زمین در صفحه دایره البروج و بردار سرعت شعاعی ستاره در صفحه دایره عظیمه گذرنده از ستاره است ، اما زاویه بین این دو بردار با زاویه بین صفحه های گذرنده از آنها برابر نیست . مثلا ممکنه دو بردار در یک صفحه باشند و با هم زاویه داشته باشند .
سوال دوم : در صورت سوال آنومالی واقعی (theta) داده شده و نیازی به حل معادله کپلر نیست .معمولا از معادله کپلر وقتی استفاده می کنیم که زمان در مسئله باشد .
برای بدست آوردن سرعت هم روش های مختلفی وجود دارد . مثلا r را بدست می آوریم و از پایستگی انرژی استفاده می کنیم . یا اندازه بردار v که در بالا به آن اشاره شده(خط دوم) و در جزوه عناصر مدار روش اثبات آن نشان داده شده است را بدست می آوریم .

اما اگر زمان سپری شده از حضیض به جای آنومالی واقعی داده شده بود ابتدا آنومالی میانگین (M=nt) را بدست می آوریم سپس باید از معادله زمان کپلر (M=E - e Sin E) آنومالی میانگین (E) را بدست آوریم وبعد آنومالی واقعی را از معادله ((( tan(theta/2)=tan(E/2)*radical((1+e)/(1-e)بدست می آوریم و بعد هم از معادلات بالا استفاده کنیم .

solh
02-04-2012, 07:55 PM
بازم با یه سوال تازه اومدم !! البته با این تفاوت که تالیف خودمه !

یه بار پرسیدم در مورد زمان برخورد دو جرم با فاصله مشخص ، و آقای shariatzadeh لطف کردن و پاسخ دادن .

اینبار مسئله سرعت هنگام برخورد دو جرم هست ، که در فاصله مشخصی از هم ساکنن ، خودم هر چی حل میکنم به یه رابطه عکس فاصله میرسم ، که با قرار دادن فاصله برخورد (تقریبا صفر ) به عدد ثابتی نمیرسه ( با حل من به بی نهایت میل میکنه ) .

poorya pvp
02-05-2012, 02:22 PM
وقتی دو جسم در حال سکون باشند و فقط نیروی گرانشی انها موثر باشه:
1:پایستگی انرژی برای لحظه ی برخورد و لحظه ی سکون(سرعت اولیه صفر)
2:پایستگی تکانه خطی برای دو حالت بالا(چون نیروی خارجی وجود ندارد پس تکانه پایسته است)
اگر روابط بالا را بنویسیم،2 معادله و 2 مجهول خواهیم داشت

shariatzadeh
02-05-2012, 07:56 PM
توی حل سوال قبل از این رابطه استفاده کردیم :vdv=adr
حالا از دو طرف انتگرال بگیرید و شتاب رو هم شتاب گرانش بزارید که به معادله پایستگی انرژی میرسید و با قرار دادن حد طرف راست از r تا 0 و قرار دادن طرف چپ از 0 تا v می تونید v رو بدست بیارید
اگر جرم ها مساوی نباشند باید از جرم کاهیده استفاده کنید

solh
02-05-2012, 09:48 PM
توی حل سوال قبل از این رابطه استفاده کردیم :vdv=adr
حالا از دو طرف انتگرال بگیرید و شتاب رو هم شتاب گرانش بزارید که به معادله پایستگی انرژی میرسید و با قرار دادن حد طرف راست از r تا 0 و قرار دادن طرف چپ از 0 تا v می تونید v رو بدست بیارید
اگر جرم ها مساوی نباشند باید از جرم کاهیده استفاده کنید

دقیقا مشکل اینه که هر چی حل میکنم اون حد سمت راستی به بینهایت میل میکنه

shariatzadeh
02-06-2012, 11:58 AM
بینهایت:crazy:
اون کاری که گفتم رو بکنی سرعت مدار سهمی بدست میاد
v=sqrt(2GM/r)
راستی توجه داشته باش که شتاب گرانش علامت منفی هم داره

solh
02-08-2012, 08:13 PM
مساحت یه مثلث کروی رو چطور حساب میکنن ؟ مستطیل چطور؟

shariatzadeh
02-08-2012, 10:01 PM
فرمول مساحت مثلث کروی عبارتست از : S=r^2(A+B+C-pi)
که S مساحت مثلث - r شعاع کره - AوBوC اندازه زاویه های مثلث کروی بر حسب رادیان است . برای اثبات این رایطه به کتاب Fundamental Astronomy مراجعه کنید .
من برای مساحت یک چهار ضلعی روی یک کره رابطه ای ندیدم اما هر چهار ضلعی رو میشه به دوتا مثلث تبدیل کرد .
دقت داشته باشید که اضلاع مثلث کروی باید دایره عظیمه باشند .

albertini
02-08-2012, 11:37 PM
سلام دوستان
ببخشيد من هم چندتا سوال درباره نجوم كروي دارم كه اگر هركدام از دوستان لطف كنند و پاسخ بدهند ممنونشان مي شوم

1 - وقتي در حل مسائل كروي كسينوس زاويه اي عددي منفي مي شود بايد همان منفي را قرار دهيم و آرك بگيريم يا قدر مطلق آنرا ؟

2- براي به دست آوردن فاصله مستقيم دو شهري كه نه طول و نه عرض جغرافيايشان يكسان نيست ، آيا بايد فرض كنيم حتمآ خط واصل آنها يك دايره عظيمه است ؟؟
اگر نه پس بايد چكار كرد ؟؟

پيشاپيش از لطف دوستان ممنونم

shariatzadeh
02-09-2012, 06:10 PM
علیک سلام
1- باید از عدد منفی آرک بگیری که آنوقت جواب در ربع سوم و یا چهارم است .
2- کمترین فاصله بین دو نقطه روی کره اندازه دایره عظیمه گذرنده از این دو نقطه است . همیشه باید فرض کنی که دایره عظیمه است ، حتی اگر عرض جغرافیایی یکسان داشتند .

Mostafa
02-09-2012, 09:29 PM
ضمن تشكر از همه دوستاني كه در اين تاپيك و تاپيك هاي مشابه المپياد نجوم زحمت مي كشند .

با توجه به ماهيت مطالب اين تاپيك ، براي اينكه با تاپيك " پرسش و پاسخ نجومي " تداخل پيدا نكند ، نام تاپيك را به " رفع اشكال المپياد نجوم " تغيير مي دهم .

براي همه اعضاي گرامي و خصوصآ دانش آموزان ، آرزوي موفقيت دارم

shokolat_g
02-11-2012, 03:43 PM
یه سوال میذارم نمیخوام که جوابشو بدین فقط توضیح بدین ممنون میشم,اگه امکانش هم هست یه سوال مشابه همین هم اگه کسی جای دیده بذاره که استفاده کنیم:

سطح زمین را با لامپ های معمولی 100w می پوشانیم. قدری ک ناظری روی ماه برای زمین اندازه میگیرد چقدر است؟؟

Astronomy
02-11-2012, 03:45 PM
یه سوال میذارم نمیخوام ک جوابشو بدین فقط توضیح بدین ممنون میشم,اگه امکانش هم هست یه سوال مشابه همین هم اگه کسی جای دیده بذاره ک استفاده کنیم:

سطح زمین را با لامپ های معمولی 100w می پوشانیم. قدری ک ناظری روی ماه برای زمین اندازه میگیرد چقدر است؟؟
چه تعداد لامپ جا میشه؟
یعنی اندازه هر لامپ 100 وات؟
یعد لامپ ها اگر در کناره های زمین باشند درخشندگی شون فرق می کنه!
قابل محاسبه هست؟

shokolat_g
02-11-2012, 03:52 PM
قانون سوم کپلر یه فرمول اصلی داره ولی تو هر کتابی ک میبینم یه شکل و یه جور نوشته:

T^2 = pi^2 /2(M+m)G

(m+M)T^2 = r^3

T^2 = 4pi^2/ GM

اگه میشه کاربراشو بگید...واحدا تو هر فرمول فرق دارن؟؟


یه سوال دیگه: r=r0/1-ecos(teta) این r0 چیه؟؟

مرسی....

shokolat_g
02-11-2012, 03:55 PM
چه تعداد لامپ جا میشه؟
یعنی اندازه هر لامپ 100 وات؟
یعد لامپ ها اگر در کناره های زمین باشند درخشندگی شون فرق می کنه!
قابل محاسبه هست؟


باید تعداد لامپا حساب بشه...توی صورت سوال نداده!!!!
بله..هر لامپ!

نمیدونم!!!!!

Astronomy
02-11-2012, 03:59 PM
باید تعداد لامپا حساب بشه...توی صورت سوال نداده!!!!
بله..هر لامپ!

نمیدونم!!!!!
منظورم اینه که اندازه هر لامپ 100 وات رو چقدر داده؟

shokolat_g
02-11-2012, 04:04 PM
منظورم اینه که اندازه هر لامپ 100 وات رو چقدر داده؟


متوجه نمیشم منظورتون از اندازه چیه !!!صورت سوالی ک نوشتم دقیقا عین سوال!!نه کم نه زیاد!

پیمان اکبرنیا
02-11-2012, 04:17 PM
منظورم اینه که اندازه هر لامپ 100 وات رو چقدر داده؟

تخمین بزنید خودتون!

Sky_Watcher
02-11-2012, 04:18 PM
اینو طبق کتاب نوشتید؟
معمولا آخر اینجور کتاب ها یه صفحه داره که ثابت هایی رو که تو کل کتاب ازشون استفاده شده(مقادیرشون) رو مینویسه.شاید اندازه ی هر لامپ 100 وات رو تو اونجا نوشته باشه.

erfan bayat
02-11-2012, 04:44 PM
اینو طبق کتاب نوشتید؟
معمولا آخر اینجور کتاب ها یه صفحه داره که ثابت هایی رو که تو کل کتاب ازشون استفاده شده(مقادیرشون) رو مینویسه.شاید اندازه ی هر لامپ 100 وات رو تو اونجا نوشته باشه.
دوستان همه‌چیو که نباید بهتون بدن از این سوالا تا دلتون بخواد توی نجوم هست. خودتون تخمین بزنید مثلا میتونید بگید هر لامپ یه دایره به شعاع۴سانتیمتر می پ.ش.نه بعد مساحتیو که میپشونه حساب کنید میتونید هر تقریبی که منطقی‌ میاد رو بزنید

Sky_Watcher
02-11-2012, 05:17 PM
آره میشه
اگه این نمونه سوالا به صورت تستی باشه معمولا فاصله ی گزینه ها از هم زیاده ، مثلا گزینه ی یک 3- هستش و گزینه ی دو 7- و از این مثالا ، چون ما طبق یه تخمین داریم سوالو حل میکنیم.
اگه غیر اینه بگید

erfan bayat
02-11-2012, 05:28 PM
قانون سوم کپلر یه فرمول اصلی داره ولی تو هر کتابی ک میبینم یه شکل و یه جور نوشته:

T^2 = pi^2 /2(M+m)G

(m+M)T^2 = r^3

T^2 = 4pi^2/ GM

اگه میشه کاربراشو بگید...واحدا تو هر فرمول فرق دارن؟؟


یه سوال دیگه: r=r0/1-ecos(teta) این r0 چیه؟؟

مرسی....
در مورد قانون سوم کپلر ببین اکثر جاها معادله به این صورت نوشته T^2 =4 pi^2 a^3 /(M+m)Gکه در اینجا همهٔ واحدهاSI است یعنی‌ زمان ثانیه و نیم قطر اطول متر و جرم کیلوگرم اگه T^2=a^3 بود که زمان بر حسب سال و نیم قطر اطول واحد نجومی یا همان au است اگه توی مخرج جمع ۲تا جرم نبود یعنی‌ از یه جرم در مقابل جرم دیگه صرف نظر شده مثل زمین- خورشید یا زمین‌ماهواره
شکلهای دیگم که نوشتی‌ یا فرمول تبدیل واحد شده در مورد معادله دومی‌ هم احتمالامی‌خواسته بگه فرموله مقاطع مخروطی به طور کلی‌ این می‌شه که به اون r0بستگی داره شما توی مسائل همون فرموله کلی بچسب به اینا زیاد توجه نکن

m-n-pour
02-11-2012, 08:00 PM
یه سوال دیگه: r=r0/1-ecos(teta) این r0 چیه؟؟
مرسی....

سلام !
برا منم سول شد ! لطفاً اگه کسی میدونه جواب بده .
من این دوتا فرمول رو دیدم :


r = a(1-e^2)/1+ecos teta
r1 = a(1-e) ---> r = r1(1-e)/1+ecos teta

shariatzadeh
02-11-2012, 09:40 PM
سه تاش یکیه
هر کتاب با توجه به کاربرد و نیازش معادله مقطع مخروطی رو یک جور می نویسه .معمولا کتاب های ریاضی به شکلی که شکلات پرسیده می نویسن یا بعضی کتاب های مکانیک مثل فولز به شکل شماره 2 می نویسن
تو کار ما چون بیشتر با کمیت هایی که به نیم قطر اطول مربوط میشه مثل دوره تناوب و انرژی سروکار داریم از شکل شماره 1 استفاده می کنیم . اما شما هر کدوم که دوست داری رو یاد بگیر چون همه اش یکیه

اما ممکنه بعضی جا ها معادله مقطع مخروطی شون تو مخرجش به جای به علاوه منها باشه ، اونجا به جای اینکه teta رو از حضیض اندازه بگیرن از اوج اندازه می گیرن . تو بعضی کتاب های مکانیک تحلیلی اینجوریه

shokolat_g
02-12-2012, 10:55 AM
اگه توی یه سوال مدت زمانی ک ربع مسیر یک ماهواره طی کرده رو بخواد میشهT tرو بدست آورد و تقسیم بر 4 کرد؟؟این سوال دارم می پرسم به خاطر اینه ک توی جواب چنین کاری رو نکرده!!

erfan bayat
02-12-2012, 11:02 AM
اگه توی یه سوال مدت زمانی ک ربع مسیر یک ماهواره طی کرده رو بخواد میشه tرو بدست آورد و تقسیم بر 4 کرد؟؟این سوال دارم می پرسم به خاطر اینه ک توی جواب چنین کاری رو نکرده!!
اینکارو نمی‌شه کرد چون طبق قانون ۲ کپلر نسبت مساحت به زمان ثابت است پس اگه گفت بود ربع مساحت مدارشو طی‌ کرده اون وقت میشد ربع دوره تناوب

اگه مدار دایره ای بود این کار درست بود چون ربع محیط همون ربع مساحته
برای حل باید از معادله کپلر(فصل5 اسمارت)استفاده کنی

shokolat_g
02-12-2012, 02:31 PM
یک ستاره شناس دو ستاره زرد رنگ را ک اعتقاد دارد دارای روشنایی های یکسان هستند آشکار میکند.ستاره Aدارای اختلاف منظر اندازه گیری شده 0.125 ثانیه قوسی است.ستارهB 400 برابر تیره تر از ستاره A آشکار میشود.فاصله تا ستاره Bچقدر است!!
سوال من اینه که B=L/4Pi d^2 این همون روشنایی؟؟پس تیره بودنش چیه؟؟ اگه توی یه سوالی Bرو بخوان ولی Lنداشته باشیم L=4(pi)R^2sigma.T^4؟؟اگه اینه R همون d؟؟
تو بعضی از کتابا F=L/4Pid^2...شار یعنی همون روشنای؟؟


تعریف ثانیه قوسی هم بدین اگه میشه :ِD:))

poorya pvp
02-12-2012, 03:25 PM
یک ستاره شناس دو ستاره زرد رنگ را ک اعتقاد دارد دارای روشنایی های یکسان هستند آشکار میکند.ستاره Aدارای اختلاف منظر اندازه گیری شده 0.125 ثانیه قوسی است.ستارهB 400 برابر تیره تر از ستاره A آشکار میشود.فاصله تا ستاره Bچقدر است!!
سوال من اینه که B=L/4Pi d^2 این همون روشنایی؟؟پس تیره بودنش چیه؟؟ اگه توی یه سوالی Bرو بخوان ولی Lنداشته باشیم L=4(pi)R^2sigma.T^4؟؟اگه اینه R همون d؟؟
تو بعضی از کتابا F=L/4Pid^2...شار یعنی همون روشنای؟؟


تعریف ثانیه قوسی هم بدین اگه میشه :ِD:))
روشنایی یعنی چگالی شار (حالا چگالی شار رو میتونیم برای هر نقطه به دست بیاریم.اگر اون نقطه،محل ناظر باشه،بهش میگن روشنایی)
شار یعنی درخشندگی
(بعضی کتابا اشتباه میکنن)
وقتی یکی 400 برابر تیره تره،یعنی اون یکی 400 برابر روشن تره
بقیش راحته دیگه
:))))))))

solh
02-12-2012, 04:45 PM
چطوری از معادله مدار داخل مختصات قطبی انتگرال بگیرم ؟

shariatzadeh
02-12-2012, 07:40 PM
این مهمترین مشکل بچه ها است که فرق شار با روشنایی چیه ؟
روشنایی یعنی مقدار انرژی عمودی رسیده به آشکارساز (ناظر ) در واحد زمان در واحد سطح : b=L/4pi*d^2
شار یعنی مقدار انرژی تولید شده توسط منبع در واحد زمان در واحد سطح منبع
: F=L/4pi*R^2
که d فاصله از ستاره و R شعاع ستاره است .
اما انتگرال معادله مدار در مختصات قطبی . شما توی کجا به این انتگرال برخورد کردی ، برای محاسبه چه چیزی ؟

shokolat_g
02-12-2012, 08:58 PM
روشنایی یعنی چگالی شار (حالا چگالی شار رو میتونیم برای هر نقطه به دست بیاریم.اگر اون نقطه،محل ناظر باشه،بهش میگن روشنایی)
شار یعنی درخشندگی
(بعضی کتابا اشتباه میکنن)
وقتی یکی 400 برابر تیره تره،یعنی اون یکی 400 برابر روشن تره
بقیش راحته دیگه
:))))))))

همونطور که آقای شریعت زاده گفتن شار با درخشندگی فرق داره!!! بله اینو میدونم که وقتی یکی 400 برابر تیره تره،یعنی اون یکی 400 برابر روشن تره ولی از طرفی گفته روشنایشون یکی جناب پوریا!!!!:d

shokolat_g
02-12-2012, 10:40 PM
این مهمترین مشکل بچه ها است که فرق شار با روشنایی چیه ؟
روشنایی یعنی مقدار انرژی عمودی رسیده به آشکارساز (ناظر ) در واحد زمان در واحد سطح : b=L/4pi*d^2
شار یعنی مقدار انرژی تولید شده توسط منبع در واحد زمان در واحد سطح منبع
: F=L/4pi*R^2
که d فاصله از ستاره و R شعاع ستاره است .
اما انتگرال معادله مدار در مختصات قطبی . شما توی کجا به این انتگرال برخورد کردی ، برای محاسبه چه چیزی ؟


این واحد d بر حسب چیه؟؟ واحد d با Rفرق نداره دیگه نه؟؟

Mah Tab
02-12-2012, 11:03 PM
شوکولات عزیز،
اولا که بله واحد d و R یکسانه!
دومـا که من فکر میکنم این که گفته روشناییشون برابره توی طول موج مرئی نیست ولی اونجایی که گفته چهارصد برابر تیره تر ( یا منظورش شاخص رنگیشه) توی طول موج مرئی داریم نگاه میکنیم.

***
برای چی میخواین از معادله ی مدار تو مختصات قطبی انتگرال بگیرید دقیقا؟D:

solh
02-13-2012, 11:24 AM
این مهمترین مشکل بچه ها است که فرق شار با روشنایی چیه ؟
روشنایی یعنی مقدار انرژی عمودی رسیده به آشکارساز (ناظر ) در واحد زمان در واحد سطح : b=L/4pi*d^2
شار یعنی مقدار انرژی تولید شده توسط منبع در واحد زمان در واحد سطح منبع
: F=L/4pi*R^2
که d فاصله از ستاره و R شعاع ستاره است .
اما انتگرال معادله مدار در مختصات قطبی . شما توی کجا به این انتگرال برخورد کردی ، برای محاسبه چه چیزی ؟

برای محاسبه نسبت فصل ها ، و یا مدت زمان عبور در قسمت مشخصی از مدار ، به روش نسبت مساحت ها

erfan bayat
02-13-2012, 11:41 AM
برای محاسبه نسبت فصل ها ، و یا مدت زمان عبور در قسمت مشخصی از مدار ، به روش نسبت مساحت ها

برای محاسبه نسبت فصلها میتونی تقریب بزنی‌ یعنی‌ به جای(ecosteta+1)^ -2 بنویسی‌ 1 منهای 2ecostetaچون خروج از مرکز مدار زمین کمه (۰/۰۱۷)تازه ضربدر یه کسینوسم شده پس میتونی از تقریب استفاده کنی‌ که دقتشم خوبه و انتگرالو بگیری ولی‌ کلا میتونی از معادله کپلر در حل این مسائل استفاده کنی‌(فصل ۵ اسمارت)
اگه جواب انتگرال هم می‌خوای به فصل ۷ ماریون مراجعه کن

poorya pvp
02-13-2012, 02:10 PM
همونطور که آقای شریعت زاده گفتن شار با درخشندگی فرق داره!!! بله اینو میدونم که وقتی یکی 400 برابر تیره تره،یعنی اون یکی 400 برابر روشن تره ولی از طرفی گفته روشنایشون یکی جناب پوریا!!!!:d

والا اقای مطهری گفتن که فرقی نمیکنه .ولی من که نگاه کردم،اقای شریعت زاده فرقی بین شار و درخشندگی بیان نکردند فقط گفتن که روشنایی و شار فرق دارن که روشنایی با درخشندگی خیلی فرق میکنه .برای برطرف شدن مشکلتون،مقاله اقای مطهری(طلا دوره 5ام) رو بخونین تا منظورمو متوجه شین(مفاهیم تابش)

poorya pvp
02-13-2012, 02:13 PM
این مهمترین مشکل بچه ها است که فرق شار با روشنایی چیه ؟
روشنایی یعنی مقدار انرژی عمودی رسیده به آشکارساز (ناظر ) در واحد زمان در واحد سطح : b=L/4pi*d^2
شار یعنی مقدار انرژی تولید شده توسط منبع در واحد زمان در واحد سطح منبع
: F=L/4pi*R^2
که d فاصله از ستاره و R شعاع ستاره است .
اما انتگرال معادله مدار در مختصات قطبی . شما توی کجا به این انتگرال برخورد کردی ، برای محاسبه چه چیزی ؟
اقای شریعت زاده،اقای مطهری به ما گفتن که شار و درخشندگی یکیه.توی مقالشون هم همینو نوشتن
ولی اونجوری که شما گفتین،شار و درخشندگی یکم فرق دارن
حالا چه کنیم؟؟؟؟؟

Mah Tab
02-13-2012, 04:52 PM
f=sigmaT^4
L=sigma T^4 4PI R^2

!!!!

پیمان اکبرنیا
02-13-2012, 07:42 PM
اقای شریعت زاده،اقای مطهری به ما گفتن که شار و درخشندگی یکیه.توی مقالشون هم همینو نوشتن
ولی اونجوری که شما گفتین،شار و درخشندگی یکم فرق دارن
حالا چه کنیم؟؟؟؟؟

در واقع شار و روشنایی هردو نشان دهنده یک مفهوم فیزیکی هستند (از لحاظ دیمانسیونی)

ولی درخشندگی کلا فرق میکند.

درخشندگی یعنی مقدار انرژی که یک ستاره در واحد زمان تولید میکند و واحد آن وات (ژول بر ثانیه) است.

اما شار و روشنایی به ترتیب مقدار انرژی تابش شده از سطح(برای سطح جسم تابنده) یا دریافت شده به سطح(برای سطح جسم دریافت کننده)، بر واحد سطح بر واحد زمان است. واحد شار و روشنایی، وات بر متر مربع یا همان ژول بر ثانیه بر متر مربع است و خیلی با درخشندگی فرق میکند.

shokolat_g
02-15-2012, 03:08 PM
شوکولات عزیز،
اولا که بله واحد d و R یکسانه!
دومـا که من فکر میکنم این که گفته روشناییشون برابره توی طول موج مرئی نیست ولی اونجایی که گفته چهارصد برابر تیره تر ( یا منظورش شاخص رنگیشه) توی طول موج مرئی داریم نگاه میکنیم.

***
برای چی میخواین از معادله ی مدار تو مختصات قطبی انتگرال بگیرید دقیقا؟D:



من فکر میکنم شما اشتباه میکنید!!!

ربطی به مرئی بودن یا نبودنش نداره!! من فکر میکنم که زمانی که گفته تیره تره منظورش شار بود!با توجه به تعریف شار(توسط منبع)

که میشه:

Ba=Bb

Fa=400Fb
160pc

اگه اشتباه بگید..:)

poorya pvp
02-15-2012, 07:43 PM
من فکر میکنم شما اشتباه میکنید!!!

ربطی به مرئی بودن یا نبودنش نداره!! من فکر میکنم که زمانی که گفته تیره تره منظورش شار بود!با توجه به تعریف شار(توسط منبع)

که میشه:

Ba=Bb

Fa=400Fb
160pc

اگه اشتباه بگید..:)
وقتی من گفتم " وقتی یکی 400 برابر تیره تره،یعنی اون یکی 400 برابر روشن تره"منظورم این بود که همون شار یا درخشندگی 400 برابر بیشتره نه روشنایی
:)

shokolat_g
02-17-2012, 06:10 PM
تفاوت های که از نظر فیزیکی ستارگان متغیر قیفاووسی با rrشلیاقی داره چیه؟؟

shariatzadeh
02-17-2012, 06:35 PM
دوره تغییر قیفاووسیان معمولا بین 1 تا 50 روز است و برونداد نوریشان در گستره 0.1 تا 2 قدر تغییر می کند . این ستارگان ستاره هایی غول پیکر اند که قطر آنها 10 تا 200 برابر قطر خورشید است . این ستارگان تپنده هستند . همچنین بین دوره تناوب و قدر مطلق میانگین شان یک رابطه وجود دارد .
ستارگان rr شلیاقی طبقه ای از ستارگان متغیراند با دوره هایی در حدود چند ساعت تا یک روز اما قدر آنها مستقل از دوره تناوبشان تقریبا صفر است .

poorya pvp
02-17-2012, 09:37 PM
دوره تغییر قیفاووسیان معمولا بین 1 تا 50 روز است و برونداد نوریشان در گستره 0.1 تا 2 قدر تغییر می کند . این ستارگان ستاره هایی غول پیکر اند که قطر آنها 10 تا 200 برابر قطر خورشید است . این ستارگان تپنده هستند . همچنین بین دوره تناوب و قدر مطلقشان یک رابطه وجود دارد .
ستارگان rr شلیاقی طبقه ای از ستارگان متغیراند با دوره هایی در حدود چند ساعت تا یک روز اما قدر آنها مستقل از دوره تناوبشان تقریبا صفر است .
میشه لطف کنید رابطشونو بگید؟؟؟؟؟؟؟؟
من جاهای مختلف،چیزای مختلف دیدم
ممنون میشم
:)

shokolat_g
02-17-2012, 11:55 PM
دوستان همه‌چیو که نباید بهتون بدن از این سوالا تا دلتون بخواد توی نجوم هست. خودتون تخمین بزنید مثلا میتونید بگید هر لامپ یه دایره به شعاع۴سانتیمتر می پ.ش.نه بعد مساحتیو که میپشونه حساب کنید میتونید هر تقریبی که منطقی‌ میاد رو بزنید

یه جای نوشته بود: Pi.D^2/4 واسه مساحت لامپ گرفته بود!من نفهمیدم چیه!! در کل اگه بدست بیاد ...L هم بدست میاد از اون طرف قدرش حساب میشه!!

shokolat_g
02-17-2012, 11:57 PM
اگه یه جسم سیاه داشته باشیم با گرم کردن سطحش تابش در طول موج های قرمز چه تغییری میکنه؟!تغییر میکنه؟

erfan bayat
02-18-2012, 10:04 AM
اگه یه جسم سیاه داشته باشیم با گرم کردن سطحش تابش در طول موج های قرمز چه تغییری میکنه؟!تغییر میکنه؟

بیشتر می‌شه چون وقتی‌ دما بالا میرود تابش در همهٔ طول موجها بیشتر میشود مطابق شکل زیر
3451

shariatzadeh
02-18-2012, 11:45 AM
میشه لطف کنید رابطشونو بگید؟؟؟؟؟؟؟؟
من جاهای مختلف،چیزای مختلف دیدم
ممنون میشم
:)
اگر نمودار قدرمطلق بر حسب دوره تناوب رو در یک مختصات نیم لوگاریتمی رسم کنیم (همون کاری که خانم هنریتا سوان لویت انجام داد) نمودار زیر به دست میاد .
این رابطه رو میتونی خودت با استفاده از نمودار زیر به دست بیاری ، کافیه معادله ی خطی که از داده ها میگذره رو حساب کنی .
دقت داشته باش که جوابت باید به شکل : M=a logP+ b باشه .
3457

Astro_M111
02-18-2012, 03:36 PM
یه سوال عدد نپر(eدرتابع پلانک) عدد ثابتیه یا نه؟

shariatzadeh
02-18-2012, 03:41 PM
بله ثابته
مقدارشم e=2.7182...
البته توی ماشین حساب هست .

COLDFIRE
02-18-2012, 10:30 PM
لطفا جواب این مسئله رو یک نفر قطعی بگه!!!! هرکی یه چیزی میگه!!!

[/URL] (http://forum.avastarco.com/forum/image/png;base64,iVBORw0KGgoAAAANSUhEUgAABAwAAAC9CAIAAAC 4WHjiAAAABmJLR0QAAAAAAAD5Q7t/AAAACXBIWXMAAA7EAAAOxAGVKw4bAAAgAElEQVR4nO3d60sU+x 8H8P0Hvk/n0T7aB/tgH+wDYUGQQISQkJAQIyIpiiKjogvlBTXKik5JpzjRBQ9F9Ysi EilEKvHYzeymbItlaWYnj5nmdde9uJfxNzN7m9mdmZ3Znb1o79 eTc1J3Ljvz/Xy/n/leRrcEAAAAAADAo8v2AQAAAAAAQG5BkgAAAAAAAAJIEgAAAAAA QABJAgAAAAAACCBJAAAAAAAAASQJAAAAAAAggCQBAAAAAAAEkC QAAAAAAIAAkgQAAAAAABBAkgAAAAAAAAJIEgAAAAAAQABJAgAA AAAACCBJAAAAAAAAASQJAAAAAAAggCQBAAAAAAAEkCQAAAAAAI AAkgQAAAAAABBAkgAAAAAAAAJIEgAAAAAAQABJAgAAAAAACCBJ AAAAAAAAASQJAAAAAAAggCQBAAAAAAAEkCQAAAAAAIAAkgQAAA AAABBAkgAAAAAAAAJIEgAAAAAAQABJAgAAAAAACCBJAAAAAAAA ASQJAGlHu76+fDnuzfZhACxvgUWnl872QcAyRy+OvXxknQ1k+z gAloGVmCQsDp3b0WhzZ/sw4LcXsNuulOuJjkHWXv/Pl9WDWfxyoSi/tj9huXAM9Qw5mIaYZ7Snb9KfiSODXJY74dRla7Tk1ebEoSxL/tneC+v0xFyfOAgsXzK3K+0aur3NyIVjnanG5sr0oQEsRyswSXD bak36LQ+n8cAJssM/8+p0MRWsjIi+5K++uRxoa7PFgqkbE7SxvF8vFhEdKfzj8b1dBu bPq9+jKv295U445e7gRDcwxKFdX25vCbWNGea63MoR/D9ur9MXnhta1GRrIrdrYN56qUwfDsfrLuREOM4JfvcC+uVynn/sf+VrTn32cP+Yf747v/zuePwdLPmL1GUwSXC8b3k8moGHqe7+ejMpOKNR0MkQ3+iNtfrC C8PLfkRK4Ne7zs8L2T6KLAo2xolxx51hFxuAA44Pf28ykMTt80 wcF1V2J1ERdPWf3NzQt8Dckde2722fQpe8wIopp4rlTjgNJglZ fgK8nOKbZ+TetmhyQEj+obZv7lxrFHq/XmAfS5gbtEldYm9X//jdcooQ8+7WkeCpeyefHi+imD2u+ft7drt2FUlfwHHZjpiJTr+t YybXbgmICky1b9YbD/U6uX/ZX+41MEXZWNUXEwQlf6GFTCUJ3uFzhUy4Mh5650z3rvzjd8qoX HtcksjC28NGot/yKAce2KXC9b6GbSGvOvdlObWi6Pkv1jE2UXePWYft2lwB33T38 UIqUkNnv3+bKxZ5jQOebB7EsrdCyqkKicOp/8ftXfsz0NXAPnKmTFXWLBajZRPfFoeaCsLPzssvW+dzOtf3zny b1Kq2lrhdA46PVysMvHBMpempq9bSF3BotvlJCv/K9Vv5d8Zl0NT6lgmu/C5+/iM/eAtTm+5P8oq05C+0kbmeBMerg5aSv//NQPK+8PagcVn1StNzQ9ZxN00veSdtH6aXQ+iSFPjVVlF0oMee7 eNQwd1fZ2ZKV9mF1jNrSMq98b6xexX6aGVEqPzqttGceHzHNnA yUCz8Y49uvJyhl+jZnpvtmeg5zJwVVE5VSBROXdZqU2YGsbB7y sRjJhnLJL55Pp/KZ5uWv3I6OUgL4e3qn2zbbuCH49Unnk0um1bx7xlwIMQ/9r9Sij8CgZ79Z4c+eCMXX/nGq1slf6GJFTgngeuWXkZJgmfgeB7RGXY///qWrW21fOK8OPbK+guxJYHAVPuWokNvF+iFt4eLKh6kkokHxxqx j+/WN39wJJMa+P67sXlbWh5yKS8W9pcHjOwIgPo+1e2xYMZFCo/d/aucyvoQq3gplIg0ltNMcduu/q36giS4b2g/7XOMjS/4mf9J9fgUHIlx/ytHmnezAoTGPBaeH87+KLEME9yuLhvX80Py9j/4V/ZJjf9n98OBdI0iS6rYrYiAAyngbl6S/8dnXhkODs7jesLu/eTXY5K/0MCySBL8jh/DI1OKH8cusyRhaWmhr3E/28cS+NV2uOaxdkME3f21Zh2xHO9ficHFNdr7RaOhQdoJJQmkoG kwqdo5MNVWodfp9Ds6ZzU/NaXFwt69xxB68GbY80L1M1NnX8OWUwPuJe/Ipe1V3fPJHWu6pFYi0lVOM8X57pDReLg3PvOjpx9u0euIYUf7R Hz9Inff2Hv2GnXUhtbhwWslhJiq1GeVarAZqGFPd64/xs8Fi4NnuOFGZO2NLC+qpoDfPjY4POHSptdDcLv6/m0uZr4G/Wb5yVXBzjBSdGkk5ruSLRhKSRa7hJZ7wIEUcDclKbo4Iuz4crz eb+SGL1fHjLuU/EXKcj1JWBw8tyrcW0iM+18qqSDSlST4J9t2cF2X1LrbY8piBjd CMmsPv7zDFwq5miL/xMDySZmUWBw8zQ7Cy8HFd1y2WjN3j6xvSapeoWc7jzU+m09Dfe AZaMwz1yT6wvzjdytKmz55mAPp2mVgYtTXNHbOyxWPwHRXlYUi 5l2t/2o3jWLllgiOd3pkQua0ApP3N1H67Z2xrQ169ulBUyjKEtPB53O C38uFU3rm0VZTybXvPt/IlWLzjn+0T235mHuzzLLrWY4lnjkq8KvzUJGemweYTPs0Q7wjl 4sjo4EMuzUIfHG3q/vjqXz2gfwLmfuGnunYXf7Hx9h7XL5gKCVV7LIiMN15KI8Q886W EcxOy2WSqxEG50XpdHErGUv+IlUZTRJ84w/2smWO6DfcUfJ8g60UKLa30Fi8eXt5gYH5HyUjUkVqNd/05+9Jjf6IsPfsM0anPSlc7iN02Uhxs+pxYloFFMdbrtFqqHySX IRLgnv4xga2diKk+OKggts1mYsTmHywyVB4/ovMRfBO9g+rqHO8E7Yhjb4ipsrZpteRwnParUmR+g285Bu9XlL Ay6rohYHm9VzSS0z7Hk+FMhqm8JSGC5mrr6q48mn67hvp4uEbv VbCm2XIRoxbwxpVatqWiFyo+IN8YzdL2e9MLzNiztl32EjW3ow JvotD3KoS0W/bXNcbHXqRQx2z7A25s2s2y0eRifimEd93thyZEj4aSI535FH71 xTGM7ETPPRc/V64vmJdgV5h/R5Cz/5TyYYvYq7p4XUpi92uvv9urGUaxvVqvwb5gqGceLFLTio3ku/732uEYbX85hByhVwUHFMgulqR6z3b7yWSP0j+IkWZShIWh0JDp iL3Z8n1BLMa2a/JWNw8orqhFR8muJ+k0AvjHT5fyK4ef6aXWyaCdg48bvua+EI4e vYVrT99ar2eWI6rWVXG9625uEC7iYDekSvFJDNP3T2Dqi9zyhd Hm80q/3P/ROuJy58TXE6un9uwV1HPl6ZHJy0w2bqxKDSudaHvqEVwnXThYw 3PqghLHHFc1ivNSUUl6eLhstWYDZuuWX/+tN3cbhJUa1RB5aWOj+N2TyrjLkVLhGeAfeLI5P9nlQ4V07icC rcdTpMMyp7P+0avluj1puCcecPu5xKPTdnLS/JP8W9fpgVTsO72mG+Jf/FJ3pHwkCxNkgS1pyN59Gt2dPKSBM/ASe6a5R3LzJDKHIpvEQH7h7aWl+NS9+zC6wNGlX3Zzt6LVxRcb 8er/UbhgERVl4N9lGJafTHJWRPRqZrcZSi9ORa6DBK3a3DwRnGzipV TEhUMFUSKXTJUBByxaMaF1fVX+36MWa/xXpzBhdX87ecf9f+Yc6d9YhEosvjlz1VEvPZlCs52vViQl/xFijKRJPi4SdqEWnPBamfuQe9Y6xYDSThOkFsgNqnB2fFhgmmO GJJvJXs+nbToy++qng5i79lf3jjgYVdqNqhZlDkw0bqBSikfpP 0x3ywT79JaUbF8Y7dKKUIVne2zc3t3DVxkWmKxDWW/y7EoPLjULo4kbmi98nO2v1D4555PJyoaFFwa9ulVgVaP8LT4jt hKubjW5vJ+ZVrIxLT30UTojvQMcU3m0P0WquiVJgmByfv1Z5) ورشید هنگام غروب کمی بیضی وار مشاهده میشود.جهت قطر بزرگ بیضی در شهر با عرض جغرافیایی 51 درجه....
به افق زاویه51 دارد
موازی افق است
به افق زاویه 39 میسازد
عمود بر افق است

[URL="http://forum.avastarco.com/forum/image/png;base64,iVBORw0KGgoAAAANSUhEUgAABAwAAAC9CAIAAAC 4WHjiAAAABmJLR0QAAAAAAAD5Q7t/AAAACXBIWXMAAA7EAAAOxAGVKw4bAAAgAElEQVR4nO3d60sU+x 8H8P0Hvk/n0T7aB/tgH+wDYUGQQISQkJAQIyIpiiKjogvlBTXKik5JpzjRBQ9F9Ysi EilEKvHYzeymbItlaWYnj5nmdde9uJfxNzN7m9mdmZ3Znb1o79 eTc1J3Ljvz/Xy/n/leRrcEAAAAAADAo8v2AQAAAAAAQG5BkgAAAAAAAAJIEgAAAAAA QABJAgAAAAAACCBJAAAAAAAAASQJAAAAAAAggCQBAAAAAAAEkC QAAAAAAIAAkgQAAAAAABBAkgAAAAAAAAJIEgAAAAAAQABJAgAA AAAACCBJAAAAAAAAASQJAAAAAAAggCQBAAAAAAAEkCQAAAAAAI AAkgQAAAAAABBAkgAAAAAAAAJIEgAAAAAAQABJAgAAAAAACCBJ AAAAAAAAASQJAAAAAAAggCQBAAAAAAAEkCQAAAAAAIAAkgQAAA AAABBAkgAAAAAAAAJIEgAAAAAAQABJAgAAAAAACCBJAAAAAAAA ASQJAGlHu76+fDnuzfZhACxvgUWnl872QcAyRy+OvXxknQ1k+z gAloGVmCQsDp3b0WhzZ/sw4LcXsNuulOuJjkHWXv/Pl9WDWfxyoSi/tj9huXAM9Qw5mIaYZ7Snb9KfiSODXJY74dRla7Tk1ebEoSxL/tneC+v0xFyfOAgsXzK3K+0aur3NyIVjnanG5sr0oQEsRyswSXD bak36LQ+n8cAJssM/8+p0MRWsjIi+5K++uRxoa7PFgqkbE7SxvF8vFhEdKfzj8b1dBu bPq9+jKv295U445e7gRDcwxKFdX25vCbWNGea63MoR/D9ur9MXnhta1GRrIrdrYN56qUwfDsfrLuREOM4JfvcC+uVynn/sf+VrTn32cP+Yf747v/zuePwdLPmL1GUwSXC8b3k8moGHqe7+ejMpOKNR0MkQ3+iNtfrC C8PLfkRK4Ne7zs8L2T6KLAo2xolxx51hFxuAA44Pf28ykMTt80 wcF1V2J1ERdPWf3NzQt8Dckde2722fQpe8wIopp4rlTjgNJglZ fgK8nOKbZ+TetmhyQEj+obZv7lxrFHq/XmAfS5gbtEldYm9X//jdcooQ8+7WkeCpeyefHi+imD2u+ft7drt2FUlfwHHZjpiJTr+t YybXbgmICky1b9YbD/U6uX/ZX+41MEXZWNUXEwQlf6GFTCUJ3uFzhUy4Mh5650z3rvzjd8qoX HtcksjC28NGot/yKAce2KXC9b6GbSGvOvdlObWi6Pkv1jE2UXePWYft2lwB33T38 UIqUkNnv3+bKxZ5jQOebB7EsrdCyqkKicOp/8ftXfsz0NXAPnKmTFXWLBajZRPfFoeaCsLPzssvW+dzOtf3zny b1Kq2lrhdA46PVysMvHBMpempq9bSF3BotvlJCv/K9Vv5d8Zl0NT6lgmu/C5+/iM/eAtTm+5P8oq05C+0kbmeBMerg5aSv//NQPK+8PagcVn1StNzQ9ZxN00veSdtH6aXQ+iSFPjVVlF0oMee7 eNQwd1fZ2ZKV9mF1jNrSMq98b6xexX6aGVEqPzqttGceHzHNnA yUCz8Y49uvJyhl+jZnpvtmeg5zJwVVE5VSBROXdZqU2YGsbB7y sRjJhnLJL55Pp/KZ5uWv3I6OUgL4e3qn2zbbuCH49Unnk0um1bx7xlwIMQ/9r9Sij8CgZ79Z4c+eCMXX/nGq1slf6GJFTgngeuWXkZJgmfgeB7RGXY///qWrW21fOK8OPbK+guxJYHAVPuWokNvF+iFt4eLKh6kkokHxxqx j+/WN39wJJMa+P67sXlbWh5yKS8W9pcHjOwIgPo+1e2xYMZFCo/d/aucyvoQq3gplIg0ltNMcduu/q36giS4b2g/7XOMjS/4mf9J9fgUHIlx/ytHmnezAoTGPBaeH87+KLEME9yuLhvX80Py9j/4V/ZJjf9n98OBdI0iS6rYrYiAAyngbl6S/8dnXhkODs7jesLu/eTXY5K/0MCySBL8jh/DI1OKH8cusyRhaWmhr3E/28cS+NV2uOaxdkME3f21Zh2xHO9ficHFNdr7RaOhQdoJJQmkoG kwqdo5MNVWodfp9Ds6ZzU/NaXFwt69xxB68GbY80L1M1NnX8OWUwPuJe/Ipe1V3fPJHWu6pFYi0lVOM8X57pDReLg3PvOjpx9u0euIYUf7R Hz9Inff2Hv2GnXUhtbhwWslhJiq1GeVarAZqGFPd64/xs8Fi4NnuOFGZO2NLC+qpoDfPjY4POHSptdDcLv6/m0uZr4G/Wb5yVXBzjBSdGkk5ruSLRhKSRa7hJZ7wIEUcDclKbo4Iuz4crz eb+SGL1fHjLuU/EXKcj1JWBw8tyrcW0iM+18qqSDSlST4J9t2cF2X1LrbY8piBjd CMmsPv7zDFwq5miL/xMDySZmUWBw8zQ7Cy8HFd1y2WjN3j6xvSapeoWc7jzU+m09Dfe AZaMwz1yT6wvzjdytKmz55mAPp2mVgYtTXNHbOyxWPwHRXlYUi 5l2t/2o3jWLllgiOd3pkQua0ApP3N1H67Z2xrQ169ulBUyjKEtPB53O C38uFU3rm0VZTybXvPt/IlWLzjn+0T235mHuzzLLrWY4lnjkq8KvzUJGemweYTPs0Q7wjl 4sjo4EMuzUIfHG3q/vjqXz2gfwLmfuGnunYXf7Hx9h7XL5gKCVV7LIiMN15KI8Q886W EcxOy2WSqxEG50XpdHErGUv+IlUZTRJ84w/2smWO6DfcUfJ8g60UKLa30Fi8eXt5gYH5HyUjUkVqNd/05+9Jjf6IsPfsM0anPSlc7iN02Uhxs+pxYloFFMdbrtFqqHySX IRLgnv4xga2diKk+OKggts1mYsTmHywyVB4/ovMRfBO9g+rqHO8E7Yhjb4ipsrZpteRwnParUmR+g285Bu9XlL Ay6rohYHm9VzSS0z7Hk+FMhqm8JSGC5mrr6q48mn67hvp4uEbv VbCm2XIRoxbwxpVatqWiFyo+IN8YzdL2e9MLzNiztl32EjW3ow JvotD3KoS0W/bXNcbHXqRQx2z7A25s2s2y0eRifimEd93thyZEj4aSI535FH71 xTGM7ETPPRc/V64vmJdgV5h/R5Cz/5TyYYvYq7p4XUpi92uvv9urGUaxvVqvwb5gqGceLFLTio3ku/732uEYbX85hByhVwUHFMgulqR6z3b7yWSP0j+IkWZShIWh0JDp iL3Z8n1BLMa2a/JWNw8orqhFR8muJ+k0AvjHT5fyK4ef6aXWyaCdg48bvua+EI4e vYVrT99ar2eWI6rWVXG9625uEC7iYDekSvFJDNP3T2Dqi9zyhd Hm80q/3P/ROuJy58TXE6un9uwV1HPl6ZHJy0w2bqxKDSudaHvqEVwnXThYw 3PqghLHHFc1ivNSUUl6eLhstWYDZuuWX/+tN3cbhJUa1RB5aWOj+N2TyrjLkVLhGeAfeLI5P9nlQ4V07icC rcdTpMMyp7P+0avluj1puCcecPu5xKPTdnLS/JP8W9fpgVTsO72mG+Jf/FJ3pHwkCxNkgS1pyN59Gt2dPKSBM/ASe6a5R3LzJDKHIpvEQH7h7aWl+NS9+zC6wNGlX3Zzt6LVxRcb 8er/UbhgERVl4N9lGJafTHJWRPRqZrcZSi9ORa6DBK3a3DwRnGzipV TEhUMFUSKXTJUBByxaMaF1fVX+36MWa/xXpzBhdX87ecf9f+Yc6d9YhEosvjlz1VEvPZlCs52vViQl/xFijKRJPi4SdqEWnPBamfuQe9Y6xYDSThOkFsgNqnB2fFhgmmO GJJvJXs+nbToy++qng5i79lf3jjgYVdqNqhZlDkw0bqBSikfpP 0x3ywT79JaUbF8Y7dKKUIVne2zc3t3DVxkWmKxDWW/y7EoPLjULo4kbmi98nO2v1D4555PJyoaFFwa9ulVgVaP8LT4jt hKubjW5vJ+ZVrIxLT30UTojvQMcU3m0P0WquiVJgmByfv1Z5"] (http://forum.avastarco.com/forum/image/png;base64,iVBORw0KGgoAAAANSUhEUgAABAwAAAC9CAIAAAC 4WHjiAAAABmJLR0QAAAAAAAD5Q7t/AAAACXBIWXMAAA7EAAAOxAGVKw4bAAAgAElEQVR4nO3d60sU+x 8H8P0Hvk/n0T7aB/tgH+wDYUGQQISQkJAQIyIpiiKjogvlBTXKik5JpzjRBQ9F9Ysi EilEKvHYzeymbItlaWYnj5nmdde9uJfxNzN7m9mdmZ3Znb1o79 eTc1J3Ljvz/Xy/n/leRrcEAAAAAADAo8v2AQAAAAAAQG5BkgAAAAAAAAJIEgAAAAAA QABJAgAAAAAACCBJAAAAAAAAASQJAAAAAAAggCQBAAAAAAAEkC QAAAAAAIAAkgQAAAAAABBAkgAAAAAAAAJIEgAAAAAAQABJAgAA AAAACCBJAAAAAAAAASQJAAAAAAAggCQBAAAAAAAEkCQAAAAAAI AAkgQAAAAAABBAkgAAAAAAAAJIEgAAAAAAQABJAgAAAAAACCBJ AAAAAAAAASQJAAAAAAAggCQBAAAAAAAEkCQAAAAAAIAAkgQAAA AAABBAkgAAAAAAAAJIEgAAAAAAQABJAgAAAAAACCBJAAAAAAAA ASQJAGlHu76+fDnuzfZhACxvgUWnl872QcAyRy+OvXxknQ1k+z gAloGVmCQsDp3b0WhzZ/sw4LcXsNuulOuJjkHWXv/Pl9WDWfxyoSi/tj9huXAM9Qw5mIaYZ7Snb9KfiSODXJY74dRla7Tk1ebEoSxL/tneC+v0xFyfOAgsXzK3K+0aur3NyIVjnanG5sr0oQEsRyswSXD bak36LQ+n8cAJssM/8+p0MRWsjIi+5K++uRxoa7PFgqkbE7SxvF8vFhEdKfzj8b1dBu bPq9+jKv295U445e7gRDcwxKFdX25vCbWNGea63MoR/D9ur9MXnhta1GRrIrdrYN56qUwfDsfrLuREOM4JfvcC+uVynn/sf+VrTn32cP+Yf747v/zuePwdLPmL1GUwSXC8b3k8moGHqe7+ejMpOKNR0MkQ3+iNtfrC C8PLfkRK4Ne7zs8L2T6KLAo2xolxx51hFxuAA44Pf28ykMTt80 wcF1V2J1ERdPWf3NzQt8Dckde2722fQpe8wIopp4rlTjgNJglZ fgK8nOKbZ+TetmhyQEj+obZv7lxrFHq/XmAfS5gbtEldYm9X//jdcooQ8+7WkeCpeyefHi+imD2u+ft7drt2FUlfwHHZjpiJTr+t YybXbgmICky1b9YbD/U6uX/ZX+41MEXZWNUXEwQlf6GFTCUJ3uFzhUy4Mh5650z3rvzjd8qoX HtcksjC28NGot/yKAce2KXC9b6GbSGvOvdlObWi6Pkv1jE2UXePWYft2lwB33T38 UIqUkNnv3+bKxZ5jQOebB7EsrdCyqkKicOp/8ftXfsz0NXAPnKmTFXWLBajZRPfFoeaCsLPzssvW+dzOtf3zny b1Kq2lrhdA46PVysMvHBMpempq9bSF3BotvlJCv/K9Vv5d8Zl0NT6lgmu/C5+/iM/eAtTm+5P8oq05C+0kbmeBMerg5aSv//NQPK+8PagcVn1StNzQ9ZxN00veSdtH6aXQ+iSFPjVVlF0oMee7 eNQwd1fZ2ZKV9mF1jNrSMq98b6xexX6aGVEqPzqttGceHzHNnA yUCz8Y49uvJyhl+jZnpvtmeg5zJwVVE5VSBROXdZqU2YGsbB7y sRjJhnLJL55Pp/KZ5uWv3I6OUgL4e3qn2zbbuCH49Unnk0um1bx7xlwIMQ/9r9Sij8CgZ79Z4c+eCMXX/nGq1slf6GJFTgngeuWXkZJgmfgeB7RGXY///qWrW21fOK8OPbK+guxJYHAVPuWokNvF+iFt4eLKh6kkokHxxqx j+/WN39wJJMa+P67sXlbWh5yKS8W9pcHjOwIgPo+1e2xYMZFCo/d/aucyvoQq3gplIg0ltNMcduu/q36giS4b2g/7XOMjS/4mf9J9fgUHIlx/ytHmnezAoTGPBaeH87+KLEME9yuLhvX80Py9j/4V/ZJjf9n98OBdI0iS6rYrYiAAyngbl6S/8dnXhkODs7jesLu/eTXY5K/0MCySBL8jh/DI1OKH8cusyRhaWmhr3E/28cS+NV2uOaxdkME3f21Zh2xHO9ficHFNdr7RaOhQdoJJQmkoG kwqdo5MNVWodfp9Ds6ZzU/NaXFwt69xxB68GbY80L1M1NnX8OWUwPuJe/Ipe1V3fPJHWu6pFYi0lVOM8X57pDReLg3PvOjpx9u0euIYUf7R Hz9Inff2Hv2GnXUhtbhwWslhJiq1GeVarAZqGFPd64/xs8Fi4NnuOFGZO2NLC+qpoDfPjY4POHSptdDcLv6/m0uZr4G/Wb5yVXBzjBSdGkk5ruSLRhKSRa7hJZ7wIEUcDclKbo4Iuz4crz eb+SGL1fHjLuU/EXKcj1JWBw8tyrcW0iM+18qqSDSlST4J9t2cF2X1LrbY8piBjd CMmsPv7zDFwq5miL/xMDySZmUWBw8zQ7Cy8HFd1y2WjN3j6xvSapeoWc7jzU+m09Dfe AZaMwz1yT6wvzjdytKmz55mAPp2mVgYtTXNHbOyxWPwHRXlYUi 5l2t/2o3jWLllgiOd3pkQua0ApP3N1H67Z2xrQ169ulBUyjKEtPB53O C38uFU3rm0VZTybXvPt/IlWLzjn+0T235mHuzzLLrWY4lnjkq8KvzUJGemweYTPs0Q7wjl 4sjo4EMuzUIfHG3q/vjqXz2gfwLmfuGnunYXf7Hx9h7XL5gKCVV7LIiMN15KI8Q886W EcxOy2WSqxEG50XpdHErGUv+IlUZTRJ84w/2smWO6DfcUfJ8g60UKLa30Fi8eXt5gYH5HyUjUkVqNd/05+9Jjf6IsPfsM0anPSlc7iN02Uhxs+pxYloFFMdbrtFqqHySX IRLgnv4xga2diKk+OKggts1mYsTmHywyVB4/ovMRfBO9g+rqHO8E7Yhjb4ipsrZpteRwnParUmR+g285Bu9XlL Ay6rohYHm9VzSS0z7Hk+FMhqm8JSGC5mrr6q48mn67hvp4uEbv VbCm2XIRoxbwxpVatqWiFyo+IN8YzdL2e9MLzNiztl32EjW3ow JvotD3KoS0W/bXNcbHXqRQx2z7A25s2s2y0eRifimEd93thyZEj4aSI535FH71 xTGM7ETPPRc/V64vmJdgV5h/R5Cz/5TyYYvYq7p4XUpi92uvv9urGUaxvVqvwb5gqGceLFLTio3ku/732uEYbX85hByhVwUHFMgulqR6z3b7yWSP0j+IkWZShIWh0JDp iL3Z8n1BLMa2a/JWNw8orqhFR8muJ+k0AvjHT5fyK4ef6aXWyaCdg48bvua+EI4e vYVrT99ar2eWI6rWVXG9625uEC7iYDekSvFJDNP3T2Dqi9zyhd Hm80q/3P/ROuJy58TXE6un9uwV1HPl6ZHJy0w2bqxKDSudaHvqEVwnXThYw 3PqghLHHFc1ivNSUUl6eLhstWYDZuuWX/+tN3cbhJUa1RB5aWOj+N2TyrjLkVLhGeAfeLI5P9nlQ4V07icC rcdTpMMyp7P+0avluj1puCcecPu5xKPTdnLS/JP8W9fpgVTsO72mG+Jf/FJ3pHwkCxNkgS1pyN59Gt2dPKSBM/ASe6a5R3LzJDKHIpvEQH7h7aWl+NS9+zC6wNGlX3Zzt6LVxRcb 8er/UbhgERVl4N9lGJafTHJWRPRqZrcZSi9ORa6DBK3a3DwRnGzipV TEhUMFUSKXTJUBByxaMaF1fVX+36MWa/xXpzBhdX87ecf9f+Yc6d9YhEosvjlz1VEvPZlCs52vViQl/xFijKRJPi4SdqEWnPBamfuQe9Y6xYDSThOkFsgNqnB2fFhgmmO GJJvJXs+nbToy++qng5i79lf3jjgYVdqNqhZlDkw0bqBSikfpP 0x3ywT79JaUbF8Y7dKKUIVne2zc3t3DVxkWmKxDWW/y7EoPLjULo4kbmi98nO2v1D4555PJyoaFFwa9ulVgVaP8LT4jt hKubjW5vJ+ZVrIxLT30UTojvQMcU3m0P0WquiVJgmByfv1Z5)

پیمان اکبرنیا
02-18-2012, 10:51 PM
لطفا جواب این مسئله رو یک نفر قطعی بگه!!!! هرکی یه چیزی میگه!!!

خورشید هنگام غروب کمی بیضی وار مشاهده میشود.جهت قطر بزرگ بیضی در شهر با عرض جغرافیایی 51 درجه....
به افق زاویه51 دارد
موازی افق است
به افق زاویه 39 میسازد
عمود بر افق است



سلام

خورشید هنگامی که به لب افق میرسه همواره به یک بیضی تبدیل میشه که قطر بزرگش موازی با افقه و مستقل از عرض جغرافیایی رصدگره.

3463

COLDFIRE
02-19-2012, 12:13 AM
دلیل علمی؟

poorya pvp
02-19-2012, 12:29 PM
دلیل علمی؟

اگه اشتباه نکنم،فصل سوم کروی تقریبا گفته چرا............

sooshans
02-19-2012, 02:50 PM
دلیل علمی؟
فک کنم این بیضی شدن خورشید بدلیل متغیر بودن اندازه شکست در ارتفاع های مختلف جو باشه خب لایه های جوم موازی افقن دیگه پس قطر اطول موازی افقه
اگه اشتباهه ببخشید

پیمان اکبرنیا
02-19-2012, 03:32 PM
فک کنم این بیضی شدن خورشید بدلیل متغیر بودن اندازه شکست در ارتفاع های مختلف جو باشه خب لایه های جوم موازی افقن دیگه پس قطر اطول موازی افقه
اگه اشتباهه ببخشید

بله به دلیل این که نوری که ارتفاع کمتری نسبت به افق داره بیشتر میشکنه، خطوط افقی طولشان ثابت میمونه ولی خطوط عمودی کوتاه میشوند.

Mah Tab
02-19-2012, 05:18 PM
کسی پاسخ هفتمین مرحله اول رو داره؟!

ali77
02-19-2012, 08:05 PM
ببخشيد يك سوال مهم

در رابطه Ma-Mb=2.5 Log (Lb/La) 1 ميخواستم ببينم براي به دست آوردن نسبت درخشندگي دو ستاره بايد در طرف چپ M ( قدر مطلق دو ستاره ) را قرار بديم يا M bol ( قدر مطلق بولومتريك دو ستاره ) ؟؟؟؟

erfan bayat
02-19-2012, 08:10 PM
ببخشيد يك سوال مهم

در رابطه Ma-Mb=2.5 Log (Lb/La) 1 ميخواستم ببينم براي به دست آوردن نسبت درخشندگي دو ستاره بايد در طرف چپ M ( قدر مطلق دو ستاره ) را قرار بديم يا M bol ( قدر مطلق بولومتريك دو ستاره ) ؟؟؟؟
اگه قدر مطلق بولومتریک بذاری درخشندگی بولومتریک بهت میده اگه قدر مطلق در یه طول موج خاص مثلا آبی بذاری درخشندگی در ناحیه‌‌ آبی بهت میده
توی سوال اگه گفتن درخشندگی یا قدر چقده منظورشون بولومتریک ولی‌ به طور کلی‌ اگه در یه طول موج خاص بخوان حتما اشاره می‌کنن مثلا یه اندیس رنگی‌ دادن شما در کل اگه چیزی ننوشته بودن خودتو درگیر نکن همینجور توی رابطه بذار

ali77
02-19-2012, 08:15 PM
خيلي ممنون كه اينقدر زود جواب دادي

پس اگر درست فهميده باشم اگر قدر مطلق بذارم درخشندگي ستاره در طول موج مرئي رو بهم ميده

و اگر قدر مطلق بولومتريك بذارم درخشندگي ستاره در تمام طول موجها رو ؟ درسته ؟

erfan bayat
02-19-2012, 08:26 PM
خيلي ممنون كه اينقدر زود جواب دادي

پس اگر درست فهميده باشم اگر قدر مطلق بذارم درخشندگي ستاره در طول موج مرئي رو بهم ميده

و اگر قدر مطلق بولومتريك بذارم درخشندگي ستاره در تمام طول موجها رو ؟ درسته ؟




ببین اگه گفت باشن قدر مطلق یا درخشندگی توی سوال بدون اشاره کردن به بازه طول موج منظورشون بولومتریک بود

نکته حواست باشه که درخشندگی بولومتریک خورشید تقریبا با مرئیش یکیه

solh
02-19-2012, 08:51 PM
کسی پاسخ هفتمین مرحله اول رو داره؟!

من داخل گوگل سرچ کردم گیرم اومد ، هر جا گشتم فقط کد 2 هم بود ، اگر کسی پاسخ کد 3 رو داره به منم بده !!

erfan bayat
02-19-2012, 08:56 PM
کسی پاسخ هفتمین مرحله اول رو داره؟!
اینجا همش هست
http://afastronomy.persianblog.ir/tag/%D8%A7%D9%84%D9%85%D9%BE%DB%8C%D8%A7%D8%AF_%D9%86% D8%AC%D9%88%D9%85

afastronomy
02-20-2012, 01:41 AM
http://afastronomy.persianblog.ir/post/157

shokolat_g
02-21-2012, 02:11 AM
وقتی میخوایم شمالی یا جنوبی بودن عرض جغرافیای توی حرکت ستاره ها بدست بیاریم چه جوری بفهمیم شمالی یا جنوبی؟!!

یه فرمولی واسه دوتای های گرفتی دیدم:L MAX=(pi.r^2F+pi.r^2F). cons tant



این consاینجا چیه؟؟

erfan bayat
02-21-2012, 07:42 AM
وقتی میخوایم شمالی یا جنوبی بودن عرض جغرافیای توی حرکت ستاره ها بدست بیاریم چه جوری بفهمیم شمالی یا جنوبی؟!!

یه فرمولی واسه دوتای های گرفتی دیدم:L MAX=(pi.r^2F+pi.r^2F). cons tant



این consاینجا چیه؟؟
در نیم کره ی شمالی ستاره ها از سمت شرق طلوع سپس به سمت جنوب میروند و در غرب غروب میکنند ولی در نیم کره ی جنوبی ستاره ها در شرق طلوع سپس به سمت شمال میروند و در غرب غروب میکنند

در رابطه ی بالا هم const یه فاصله و حساسیت دستگاه گیرنده و جو بستگی دارد
اگه اطلاعات بیشتر میخای صفحه ی 88-90 جلد1 اخترفیزیک ستاره ای ببین

andromeda-s
02-21-2012, 04:09 PM
حداقل و حداکثر زاویه مثلث کروی با ضلعش چیه؟؟

erfan bayat
02-21-2012, 05:46 PM
حداقل و حداکثر زاویه مثلث کروی با ضلعش چیه؟؟
اگه منظورت ماکزیمم برای یک ضلع یا زاویه مثلث کروی برای یک ضلع بین0و180
برای زاویه هم مثل ضلع 0-180

andromeda-s
02-21-2012, 07:07 PM
یه سوال اگر یه مثلث کروی متساوی الاضلاع داشته باشیم و زاویش میل کنه به صفر ضلعشم میل میکنه به صفر؟ اگر ضلع میل کنه به صفر زاویه میشه پی سوم اینو میدونم درسته ولی نمی تونم درکش کنم چه جوری میشه؟؟

shariatzadeh
02-21-2012, 09:45 PM
یه سوال اگر یه مثلث کروی متساوی الاضلاع داشته باشیم و زاویش میل کنه به صفر ضلعشم میل میکنه به صفر؟ اگر ضلع میل کنه به صفر زاویه میشه پی سوم اینو میدونم درسته ولی نمی تونم درکش کنم چه جوری میشه؟؟
سوال1: از رابطه کسینوس ها نتیجه میشه که کسینوس اون ضلع برابر با کسینوس تفاضل دو ضلع دیگه که چون مثلث متساوی الاضلاع هست پس صفر میشه
سوال2 : مساحت مثلث کروی به صفر میل میکنه ، از رابطه مساحت مثلث کروی نتیجه می گیریم مجموع زاویه ها به سمت پی میل میکنه پس هر زاویه به پی سوم میل میکنه .

Amirali
02-22-2012, 02:00 PM
ماهواره ای در مداری دایره ای به ارتفاع 600 کیلومتر از از سطح زمین و با انحراف مداری 28درجه قرار دارد. برای اینکه به مداری با همان اندازه و انحراف مداری 20درجه برود سرعتش چقدر تغییر کند؟

ماهواره ای را به روش انتقال مداری هوهمان در مداری دایره ای به ارتفاع 7000 کیلومتر از سطح زمین انتقال داده ایم. این ماهواره چند ساعت پس از قرار گرفتن در مدار در نقطه پرتابش ظاهر می شود؟

جسمی با سرعت 30000 متر بر ثانیه با پارامتر برخورد 5/2 واحد نجومی به سمت منظومه شمسی در حال حرکت است. این جسم بر اثر گرانش خورشید چند درجه از مسیر خود منحرف میشود؟

erfan bayat
02-22-2012, 02:45 PM
ماهواره ای در مداری دایره ای به ارتفاع 600 کیلومتر از از سطح زمین و با انحراف مداری 28درجه قرار دارد. برای اینکه به مداری با همان اندازه و انحراف مداری 20درجه برود سرعتش چقدر تغییر کند؟

ماهواره ای را به روش انتقال مداری هوهمان در مداری دایره ای به ارتفاع 7000 کیلومتر از سطح زمین انتقال داده ایم. این ماهواره چند ساعت پس از قرار گرفتن در مدار در نقطه پرتابش ظاهر می شود؟

جسمی با سرعت 30000 متر بر ثانیه با پارامتر برخورد 5/2 واحد نجومی به سمت منظومه شمسی در حال حرکت است. این جسم بر اثر گرانش خورشید چند درجه از مسیر خود منحرف میشود؟

سوال اول:فقط لازمه یک جمع برداری در نقطه ی گره ی دو مدار (استوا)بنویسی که یک مثلث تشکیل میشه که دو ضلعش سرعت در مدار دایره ای و زاویه ی بینشون i و ضلع سوم همان تغییر سرعت است که با یک قانون cos میزان تغییر سرعت بدست میاد
سوال2:باید زمین یک مسیر دایره ای و مدار رو یک مسیر دایره ای دیگه در نظر بگیری که بینشون میخایم با یک بیضی انتقال مداری انجام بدیم (که اینم تو استوا چون سرعت زمین تو استوا بیشتره و با اختلاف سرعت کمتری میتونیم وارد مدار دایره ایش کنیم که بهینه تره ) حالا قطر اطول بیضی میشه شعاع زمین + شعاع مدار دایروی پس دوره تناوبش از کپلر بدست میاد که نصف این دوره تناوب طول میکشه تا به مدار دایره ای برسه که زمینم تو این مدت یه مقدار میچرخه.حالا اختلاف زاویه ای ناظر و جسم در مدار دایروی را داریم که با بدست اوردن سرعت زاویه ای نسبی زمین و مدار دایروی میتونیم مدت زمان رو بدست بیاریم
سوال3:چون جسم در فاصله ی خیلی دور سرعت داره پس مدار هذلولوی است ابتدا تکانه زاویه ای بر واحد جرم جسم و انرژی بر واحد جرم جسم را برای بینهایت مینویسی (حواست باشه که انرژی تو بینها یت فقط جنبشیه همینطور تکانه زاویه ای بر واحد جرم میشه سرعت *پارامتر برخورد) نیم قطر اطول مدار که از انرژی بدست میاد و از تکانه زاویه ای خروج از مرکز هذلولی بدست میاد(امیدوارم روابطو بلد باشی)
حالا از رابطه ی قطبی برای هذلولی r برابر بینهایت بذار که یه رابطه بین eوتتا بدست میاد که 180منهای 2تتا جواب مسئله است
امیدوارم فهمیده باشی

Mah Tab
02-22-2012, 10:45 PM
یه سوال.
یه مقایسه سن جهان توی هندسه های مختلف رو میشه یکی اینجا بنویسه؟
بعدم شتاب انبساط مثبته دیگه؟!
مرسی.

solh
02-23-2012, 09:46 AM
یه سوال.
یه مقایسه سن جهان توی هندسه های مختلف رو میشه یکی اینجا بنویسه؟
بعدم شتاب انبساط مثبته دیگه؟!
مرسی.

اختر فیزیک مقدماتی رو اگه داری ، فصل آخرش ، توضیح داده . اره شتاب مثبته ( البته احتمالا !، کیهانشناسیه دیگه ،
یه مرتبه دیدی نبود !)

solh
03-18-2012, 08:22 PM
داشتم جلد سوم اختر فیزیک ستاره ای رو میخوندم ، در فصل مربوط به منابع انرژی ، بعد از کمی تجزیه و تحلیل انرژی حاصل از سوخت هیدروژن و هلیوم و...(صفحه ی 115 )، این جور مینویسه : بنابراین ستاره ای با جرم و تابندگی معین ، با این منبع انرژی حد اکثر 10 درصد زمانی که منبع آن از سوخت هیدروژن تامین میشود میتواند دوام بیاورد .

میشه کمی توضیح بدید، مثلا چجوری ؟

erfan bayat
03-18-2012, 09:19 PM
داشتم جلد سوم اختر فیزیک ستاره ای رو میخوندم ، در فصل مربوط به منابع انرژی ، بعد از کمی تجزیه و تحلیل انرژی حاصل از سوخت هیدروژن و هلیوم و...(صفحه ی 115 )، این جور مینویسه : بنابراین ستاره ای با جرم و تابندگی معین ، با این منبع انرژی حد اکثر 10 درصد زمانی که منبع آن از سوخت هیدروژن تامین میشود میتواند دوام بیاورد .

میشه کمی توضیح بدید، مثلا چجوری ؟
تو متن گفته انرژی حاصل از واکنش هر He در واکنش p-p نسبت به 3alfa برابر 10 است
ستاره تا یک شعاعی همجوشی انجام میده که اولش پر از هیدروژن بوده که در طی عمر رشته اصلی این هسته ی همجوشی ,هلیمی میشه
حالا گفته دو تا ستاره با تابندگی یکسان را در نظر بگیریم که یکی همجوشی هیدروژن و دیگری3alfa
چون گفته انرژی حاصل از واکنش هر He در واکنش p-p نسبت به 3alfa برابر 10 است و از طرفی تابندگی یکسانه و جرم یکسانه و مقدار سوخت یکی میشه پس باید عمر ستاره 3alfa نسبت به p-p برابر 1/10

solh
03-23-2012, 05:00 PM
طول دایرت البروجی حضیض رو چطور بدست بیارم ؟

درسته که زاویه ای که خورشید با حضیض میسازه رو به اضافه ی طول خورشید کنم ؟

shariatzadeh
03-23-2012, 05:46 PM
طول دایرت البروجی حضیض رو چطور بدست بیارم ؟

درسته که زاویه ای که خورشید با حضیض میسازه رو به اضافه ی طول خورشید کنم ؟

طول دایره البروجی حضیض یا آرگومان حضیض که یکی از عناطر مدار نیز محسوب می شود عبارتست از :

ω= λ -θ

که در آن λ طول دایره البرجی خورشید و θ زاویه ای است که زمین با حضیض مدار خودش میسازد (آنومالی واقعی)

solh
03-30-2012, 06:40 PM
یکی از سوالات دورهی سوم ، مرحله دو بود (زیر 15 سال ) ، چجوری حل میشه ؟

...ناظری در نقطه ای با طول و عرض جغرافیایی (0,0)ایستاده است و با دست به سوی نقطه ای روی زمین اشاره میکند به طوری که سمت و ارتفاع دست وی به ترتیب A و a هستند (a<0) . اگر بتوانیم در امتداد دست این فرد تونلی مستقیم حفر کنیم ،دهانه ی دیگر تونل در چه طول و عرض جغرافیایی قرار میگیرد ؟

Astronomer
03-30-2012, 06:49 PM
یکی از سوالات دورهی سوم ، مرحله دو بود (زیر 15 سال ) ، چجوری حل میشه ؟

...ناظری در نقطه ای با طول و عرض جغرافیایی (0,0)ایستاده است و با دست به سوی نقطه ای روی زمین اشاره میکند به طوری که سمت و ارتفاع دست وی به ترتیب A و a هستند (a<0) . اگر بتوانیم در امتداد دست این فرد تونلی مستقیم حفر کنیم ،دهانه ی دیگر تونل در چه طول و عرض جغرافیایی قرار میگیرد ؟
کمی هندسه وارد بحث کنیم مسئله حله...
ما یک مثلث کروی داریم روی زمین(دوسر تونل و قطب شمال) و یک مثلث مسطحه که از مرکز زمین و دو سر تونل تشکیل میشه...
خوب با کمی هندسه میشه زاویه ها رو پیدا کرد مثلا زاویه بین دایره عظیمه دو سر تونل با نصف النهار ناظر میشه همون زاویه سمت همینطور زاویه ضلع مسطح دوسر تونل با خط فرضی رساننده سر تونل به مرکز زمین هم میشه 180منهای ارتفاع!!! از طرفی مثلث مسطحمون متساوی الساقین هست پس ...
بقیه رو خودتون برید دیگه... زاوایا پیداشد و موندنوشتن روابط بین اونها...

albertini
04-16-2012, 09:25 PM
سلام به اعضا و مدیران محترم آوااستار
من از دیروز مطالعه کتاب فیزیک ستاره ها برا مبتکران رو شروع کردم
از همون اول که شروع کردم سوالاتی برای پیش اومد که هرچی گشتم جوابشو پیدا نکردم
نمیدونم سوالاتم رو کجا باید مطرح کنم ولی اگر میشه همینجا بپرسم
قبلش عذر خواهی می کنم که شاید سوالاتم ابتدایی باشن :

1- در جاهایی از کتاب برای دما از واحد MeV استفاده شده . هرچی گشتم نفهمیدم Mev یعنی چی ؟
2- در فرمول های ریاضی [ ] exp یعنی چی ؟؟

فعلا همینا

ممنون میشم اگر دوستان راهنماییم کنند

shariatzadeh
04-16-2012, 09:45 PM
1- در جاهایی از کتاب برای دما از واحد MeV استفاده شده . هرچی گشتم نفهمیدم Mev یعنی چی ؟
2- در فرمول های ریاضی [ ] exp یعنی چی ؟؟



1- توی بعضی از کتاب ها به جای دما از K*T استفاده می کنن که واحدش انرژِیه (K ثابت بولتزمان هست) .
همچنین این مقدار رو بر حسب الکترون ولت یا eV بیان می کنن که 1eV برابر با 19-^10*1.6 هست و 1MeV هم برابر با 6^10 eV .

2- exp [x]=e^x که e عدد نپر نامیده میشه و مقدارش تقریبا 2.73 هست .توی ماشین حساب های مهندسی این عبارت وجود داره .

ata moradi
04-24-2012, 12:35 AM
سلام من احساس کردم که بچه ها قبل از مرحله 2 به تمرین بیشتری احتیاج دارند ، به همین خاطر یه سوال مکانیک با حلشو نوشتم امیدوارم مفید واقع بشه.


دانلود حل مثال (http://s2.picofile.com/file/7363353973/1%D8%AD%D9%84_%D9%85%DA%A9%D8%A7%D9%86%DB%8C%DA%A9 .pdf.html)

ata moradi
04-24-2012, 08:45 PM
این هم یه سوال دیگه که بازم مکانیکه با حلش که برای تمرین مرحله 2 خوبه.

دانلود مثال مکانیک (http://s2.picofile.com/file/7363384836/%D8%AD%D9%84_%D9%85%D8%AB%D8%A7%D9%84.pdf.html)

andromeda-s
04-29-2012, 04:43 PM
میشه فقط جواب آخر سوال دو(همون ایستگاه فضاییه)مرحله دو هفتمین دوررو بدید مرسی:)

shariatzadeh
04-29-2012, 07:07 PM
میشه فقط جواب آخر سوال دو(همون ایستگاه فضاییه)مرحله دو هفتمین دوررو بدید مرسی:)

با توجه به اینکه مدت زمان پرواز داده شده می توان نتیجه گرفت زاویه ای که فضانورد و ایستگاه طی می کنند میشه 120 درجه .
حالا با استفاده از قانون دوم کپلر که نسبت مساحت جاروب شده به مدت زمان گذشته برابر با نصف تکانه زاویه ای ویژه است و h=sqrt(2GMa(1-e^2)) . حال برای مدار دایروی و یکبار برای مدار بیضی این دو معادله رو بنویس و به هم تقسیم کن تا یک طرف نسبت مساحت ها و طرف دیگر نسبت sqrt(a(1-e^2)/R) باشه که a نیم قطر اطول مدار بیضی و R شعاع مدار دایروی هست .
دقت کن که مساحتی که ایستگاه فضایی جاروب میکنه برابر با 4/3مساحت دایره هست و مساحتی که فضانورد جاروب میکنه 0.728*pi*r^2 + ثلث مساحت دایره هست .
خوب حالا نسبت a/R رو هم از معادله مدار جایگذاری می کنیم و زاویه تتا هم برابره 120 درجه هست. حالا اعداد رو جایگذاری کن و ساده کن تا به e=0.73 برسی .

sooshans
04-30-2012, 10:26 AM
سلام عرض میکنم خدمت دوستان
تو حل سوال سوم مرحله دو هفتمین دوره به مشکل برخوردم عجیب!
اگه میشه یه نفر جواب آخر هر دو قسمت رو بگه!
مرسی:thumbsup:

solh
04-30-2012, 10:39 AM
سلام عرض میکنم خدمت دوستان
تو حل سوال سوم مرحله دو هفتمین دوره به مشکل برخوردم عجیب!
اگه میشه یه نفر جواب آخر هر دو قسمت رو بگه!
مرسی:thumbsup:

چون تابش باعث میشه یه نیرو برعکس گرانش وراد بشه ، باعث میشه شتاب کند شونده بشه ، یعنی مسئله پیدا کردن نقطه ی عطفه ، توی نمودار اونجایی که تقعر عوض میشه در واقع جاییه که شتاب از تند شوندگی خودش جلوگیری میکنه ، تقریبا میشه 4000sفکر کنم ، در مورد مسئله ی دوم هم توی نقطه ی عطف شتاب صفره ، پس نیروهای تابشی و گرانشی برابرند که با جایگذاری مساحت بدست میاد . من سو استفاده کردم جوابو گفتم که اشکالش گرفته بشه !دوستان لطف کنن تصحیح کنن .

یه سوال هم دارم ، توی رابطه ی عکس مجزوریه نیروی تابشی فاصله از سطح ستاره حساب میشه و توی گرانش فاصله از مرکز دیگه ؟ ( آخه یکی از دوستان توی یه مسئله دیگه اومده بودن با مساوی قرار دادن این دو نیرو فاصله ها رو خط زده بودند)

sooshans
04-30-2012, 10:48 AM
چون تابش باعث میشه یه نیرو برعکس گرانش وراد بشه ، باعث میشه شتاب کند شونده بشه ، یعنی مسئله پیدا کردن نقطه ی عطفه ، توی نمودار اونجایی که تقعر عوض میشه در واقع جاییه که شتاب از تند شوندگی خودش جلوگیری میکنه ، تقریبا میشه 4000sفکر کنم ، در مورد مسئله ی دوم هم توی نقطه ی عطف شتاب صفره ، پس نیروهای تابشی و گرانشی برابرند که با جایگذاری مساحت بدست میاد . من سو استفاده کردم جوابو گفتم که اشکالش گرفته بشه !دوستان لطف کنن تصحیح کنن .

یه سوال هم دارم ، توی رابطه ی عکس مجزوریه نیروی تابشی فاصله از سطح ستاره حساب میشه و توی گرانش فاصله از مرکز دیگه ؟ ( آخه یکی از دوستان توی یه مسئله دیگه اومده بودن با مساوی قرار دادن این دو کمیت فاصله ها رو خط زده بودند)
منم همین کارو کردم و همین حدود و برا الف بدست آوردم ولی خب دقیق نمیشه این عدد و تعیین کرد! وقتی میخوان تصحیح کنن چند درصد خطا برا این سوال در نظر میگیرن؟ باز اگه نقطه اوج قبلی نمودار و میداد میشد یه کاریش کرد!
حالا اینا به کنار چرا نیرو ها برابر باشن؟ خب یکی از اون یکی بیشتر باشه که مشکلی پیش نمیاد شتاب جهتش عوض میشه! نیرو ها در هیچ کجای مسیر نمیتونن مساوی بشن!
مرسی

solh
04-30-2012, 10:52 AM
منم همین کارو کردم و همین حدود و برا الف بدست آوردم ولی خب دقیق نمیشه این عدد و تعیین کرد! وقتی میخوان تصحیح کنن چند درصد خطا برا این سوال در نظر میگیرن؟ باز اگه نقطه اوج قبلی نمودار و میداد میشد یه کاریش کرد!
حالا اینا به کنار چرا نیرو ها برابر باشن؟ خب یکی از اون یکی بیشتر باشه که مشکلی پیش نمیاد شتاب جهتش عوض میشه! نیرو ها در هیچ کجای مسیر نمیتونن مساوی بشن!
مرسی

چون تقعر عوض شده لاجرم باید نیروها توی نقطه ی عطف صفر بشه ! البته حالا که فکر میکنم میبینم یه ابهامی هم وجود داره چون توی همون نقطه بادبان باز شده ! ولی شاید توی نقطه ای که سرعت صفره بتونیم جایگذاری کنیم .

sooshans
04-30-2012, 10:58 AM
چون تقعر عوض شده لاجرم باید نیروها توی نقطه ی عطف صفر بشه !
من دلیل موجهی برا این حرفتون نمی بینم!
خب شما فرض کنین داره با یه نیرویی میره یه سمتی یه دفه یه نیروی خلاف جهت خیلی بزرگتر قبلی بهش وارد میشه! خب جهت شتاب عوض میشه ولی به مسیرش تا یه جایی که در این سوال اون نقطه مینیممه ادامه میده!
ضمنا اگه نیروها در یه نقطه برابر بشن چون تو هر دو نیرو آر به توان منفی 2 داریم تاثیر آر از بین میره و همه جا نیرو ها مساوی میشن!

solh
04-30-2012, 11:02 AM
من دلیل موجهی برا این حرفتون نمی بینم!
خب شما فرض کنین داره با یه نیرویی میره یه سمتی یه دفه یه نیروی خلاف جهت خیلی بزرگتر قبلی بهش وارد میشه! خب جهت شتاب عوض میشه ولی به مسیرش تا یه جایی که در این سوال اون نقطه مینیممه ادامه میده!
ضمنا اگه نیروها در یه نقطه برابر بشن چون تو هر دو نیرو آر به توان منفی 2 داریم تاثیر آر از بین میره و همه جا نیرو ها مساوی میشن!

خوب نیست دیگه اون ار کجا این ار کجا !! توی بینهایت آره ولی الان اون اره همونطور که گفتم از سطحه اون یکی از مرکز .

الان که بیشتر نگاه کردم سمت راست نمودار باز هم توی همون فاصله نقطه عطف داریم .

sooshans
04-30-2012, 11:07 AM
خوب نیست دیگه اون ار کجا این ار کجا !! توی بینهایت آره ولی الان اون اره همونطور که گفتم از سطحه اون یکی از مرکز .
فک نکنم منظور سوال این باشه چون اصلا درباره شعاع غول حرفی نزده ما هم معمولا برای شار دریافتی فاصله رو از مرکز حساب می کنیم!

solh
04-30-2012, 11:16 AM
فک نکنم منظور سوال این باشه چون اصلا درباره شعاع غول حرفی نزده ما هم معمولا برای شار دریافتی فاصله رو از مرکز حساب می کنیم!

خوب شاید چون هم جرم خورشیده ، بتونیم دماش رو کمی کمتر از خورشید گرفته شعاعش رو حساب کنیم .

sooshans
04-30-2012, 11:19 AM
خوب شاید چون هم جرم خورشیده ، بتونیم دماش رو کمی کمتر از خورشید گرفته شعاعش رو حساب کنیم .
جرم؟ دما؟ چجوری این تقریبو زدین؟ با چه رابطه ای؟

solh
04-30-2012, 11:23 AM
جرم؟ دما؟ چجوری این تقریبو زدین؟ با چه رابطه ای؟

معمولا غول سرخ ها ی هم رده طیفی دماشون 300 ، 400 تا از ستاره رشته اصلی کمتره ، البته این طور که توی جدول ها دیدم ، ولی شاید نیازی به این کارا نباشه و به قول خودت فرقی نمیکنه توی چه فاصله ای باشه و بتونیم فاصله ها رو خط بزنیم ، اونوقت اصلا مهم نیست توی چه فاصله ای نیرو ها صفر میشن ، و راحت حل بشه ولی با توجه به این که در مقایسه با شعاع غول فاصله کمه فر نمیکنم کار درستی باشه . دوسِتان کمک کنید !

sooshans
04-30-2012, 11:28 AM
معمولا غول سرخ ها ی هم رده طیفی دماشون 300 ، 400 تا از ستاره رشته اصلی کمتره ، البته این طور که توی جدول ها دیدم ، ولی شاید نیازی به این کارا نباشه و به قول خودت فرقی نمیکنه توی چه فاصله ای باشه و بتونیم فاصله ها رو خط بزنیم ، اونوقت اصلا مهم نیست توی چه فاصله ای نیرو ها صفر میشن ، و راحت حل بشه .
این جملتون نابودم کرد! :grin:حل نمیشه اصلا چه برسه به راحت!
وقتی تو یه نقطه مساوی باشن همه جا مساوی ان و ما فقط یه جسم با یه سرعت ثابت داریم که به سمت غول خواهد رفت و در آن غرق خواهد شد(چه سرنوشت تاسف باری!)
جدا از شوخی فک نکنم راه مساله این باشه!
باید دنبال یه چیز جدید بود! مساوی گذاشتن نیرو ها جواب نمیده!:thumbsup:

solh
04-30-2012, 11:32 AM
این جملتون نابودم کرد! :grin:حل نمیشه اصلا چه برسه به راحت!
وقتی تو یه نقطه مساوی باشن همه جا مساوی ان و ما فقط یه جسم با یه سرعت ثابت داریم که به سمت غول خواهد رفت و در آن غرق خواهد شد(چه سرنوشت تاسف باری!)
جدا از شوخی فک نکنم راه مساله این باشه!
باید دنبال یه چیز جدید بود! مساوی گذاشتن نیرو ها جواب نمیده!:thumbsup:

با توجه به چند نکته فکر کنم حل بشه : 1. علاوه بر سمت چپ نمودار سمت راست نمودار هم توی تقریبا همون فاصله نقطه عطف داریم ، پس اونجا نیروها صفر شدن 2. فاصله از ابرغول در مقایسه با شعاع ابر غول زیاد نیست پس نمیشه از شعاع صرف نظر کرد .

erfan bayat
04-30-2012, 02:26 PM
با توجه به چند نکته فکر کنم حل بشه : 1. علاوه بر سمت چپ نمودار سمت راست نمودار هم توی تقریبا همون فاصله نقطه عطف داریم ، پس اونجا نیروها صفر شدن 2. فاصله از ابرغول در مقایسه با شعاع ابر غول زیاد نیست پس نمیشه از شعاع صرف نظر کرد .
جواب قسمت اول درسته ولی قسمت 2 نیرو ها برابر نمیشه هاااااااااااااااااااا
سرعت صفر میشه مثلا یه جسم که با سرعت میره ما نیرو وارد میکنیم خلاف جهتش همینجوری. یه جا سرعت صفر میشه ولی نیرو نه!!
اینم جواب (http://erfan007.persiangig.com/BOOKS/1.jpg)

sooshans
04-30-2012, 03:36 PM
سلام دوباره
طول حضیض=100 درجه یعنی چی؟

shariatzadeh
04-30-2012, 05:35 PM
سلام دوباره
طول حضیض=100 درجه یعنی چی؟

سلام
یعنی زاویه ای که حضیض مدار با نقطه اعتدال بهاری میسازه برابر با 100 درجه است . درواقع طول سماوی نقطه حضیض برابر با 100 درجه است .

erfan bayat
04-30-2012, 08:17 PM
جواب سوال 5 مرحله 2 90 (http://erfan007.persiangig.com/BOOKS/2.jpg)

albertini
05-07-2012, 12:21 PM
ببخشيد اساتيد

بازم يك سوال ابتدايي دارم كه نتونستم جوابش رو پيدا كنم

در بعضي از كتابها به عبارت هايي بر ميخورم كه مثلا حرف r رو يا يك يا دونقطه بالاي اون نوشته ، اينجوري : http://up.avastarco.com/images/dbkql7z23f1jazkjvk.jpg (http://up.avastarco.com/)

اولا تلفظ اين دوتا رو چي بايد بگيم ؟

و مهمتر از اون ، كاربرد اينها چي هست و چه مفهومي دارن ؟؟

اگر منبعي هم براي مطالعه اين موضوع هست لطفا معرفي كنيد

خيلي از همتون متشكرم

khaste
05-07-2012, 01:00 PM
ببخشيد اساتيد

بازم يك سوال ابتدايي دارم كه نتونستم جوابش رو پيدا كنم

در بعضي از كتابها به عبارت هايي بر ميخورم كه مثلا حرف r رو يا يك يا دونقطه بالاي اون نوشته ، اينجوري : http://up.avastarco.com/images/dbkql7z23f1jazkjvk.jpg (http://up.avastarco.com/)


اولا تلفظ اين دوتا رو چي بايد بگيم ؟

و مهمتر از اون ، كاربرد اينها چي هست و چه مفهومي دارن ؟؟

اگر منبعي هم براي مطالعه اين موضوع هست لطفا معرفي كنيد

خيلي از همتون متشكرم

اونی که یه دونه نقطه داره r دات خونده میشه یعنی یک باز ازش مشتق گرفتیم
وقتی از مکان نسبت به زمان مشتق بگیریم سرعت رو به ما میده که معمولا r بردار مکان هست
وقتی 2تا نقطه داره یعنی 2بار مشتق گرفتیم
فکر کنم اول مکانیک فولز توضیح داده + موارده استفاده

A.mousavi
05-10-2012, 04:05 PM
ببخشيد اساتيد

بازم يك سوال ابتدايي دارم كه نتونستم جوابش رو پيدا كنم

در بعضي از كتابها به عبارت هايي بر ميخورم كه مثلا حرف r رو يا يك يا دونقطه بالاي اون نوشته ، اينجوري : http://up.avastarco.com/images/dbkql7z23f1jazkjvk.jpg (http://up.avastarco.com/)

اولا تلفظ اين دوتا رو چي بايد بگيم ؟

و مهمتر از اون ، كاربرد اينها چي هست و چه مفهومي دارن ؟؟

اگر منبعي هم براي مطالعه اين موضوع هست لطفا معرفي كنيد

خيلي از همتون متشكرم

وقتی که دوبار ازش مشتق بگیری ، در واقع شتاب رو بدست میاری
یکی از کاربردای دات خلاصه کردنه که با این کار فرمول ها رو مبتونی ساده تر کنی و بفهمی که تو فرمول چه اتفاقاتی داره می افته.
اگه وقت داری یه سر به کتاب فیزیک کلپنر بزن.
همشو متوجه میشی.

albertini
08-29-2012, 10:54 PM
سلام

ببخشيد يه مساله هست كه من حل كردم ولی نمدونم درسته يا نه

ميخواستم از اساتيد خواهش كنم جوابم رو چك كنند

صورت : يك نفر ميخواد از شهری در عرض جغرافيايی 65 و طول 20 درجه ، بره به شهري در عرض جغرافيايی 65 و طول 80 درجه .

اين شخص ميخواد از كوتاهترين مسير بره

مختصات اين فرد رو در زمانی كه در وسط مسير قرار داره تعيين كنيد

جواب من اينه كه چون عرض جغرافياييش تغيير نمي كنه خب وسط مسير ميشه همان عرض جغرافيايی و نصف اختلاف طول دو شهر يعنی مختصاتش در وسط مسير

ميشه : عرض 65 درجه و طول 50 درجه

درست گفتم ؟

Mostafa
08-29-2012, 11:40 PM
سلام

ببخشيد يه مساله هست كه من حل كردم ولی نمدونم درسته يا نه

ميخواستم از اساتيد خواهش كنم جوابم رو چك كنند

صورت : يك نفر ميخواد از شهری در عرض جغرافيايی 65 و طول 20 درجه ، بره به شهري در عرض جغرافيايی 65 و طول 80 درجه .

اين شخص ميخواد از كوتاهترين مسير بره

مختصات اين فرد رو در زمانی كه در وسط مسير قرار داره تعيين كنيد

جواب من اينه كه چون عرض جغرافياييش تغيير نمي كنه خب وسط مسير ميشه همان عرض جغرافيايی و نصف اختلاف طول دو شهر يعنی مختصاتش در وسط مسير

ميشه : عرض 65 درجه و طول 50 درجه

درست گفتم ؟


سلام آلبرتيني عزيز

فكر مي كنم جوابتون متآسفانه درست نباشه

اگر دقت كنيد در سوال گفته كه بايد كوتاه ترين فاصله طي شود

اما كوتاهترين فاصله بين دو نقطه بر روي يك مدار ( نقاطي با عرض جغرافيايي يكسان ) امتداد مدار واصل بين آن دو نقطه نيست

كوتاهترين فاصله بين دو نقطه روي كره ، دايره عظيمه اي است كه از آن دو نقطه عبور مي كند و مدار ها هم كه دايره عظيمه نيستند ( بجز استوا )

بنابر اين اين شخص اگر بخواهد كوتاهترين فاصله بين اين دو شهر را طي كند نبايد در امتداد مدار 65 درجه حركت كند و در نتيجه مختصات وسط مسير هم متفاوت با چيزي است

كه شما گفتيد

اما براي حل بايد ابتدا دايره عظيمه بين اين دو شهر را رسم كنيد و سپس نصف النهار هاي گذرنده از اين دو شهر را رسم كنيد ( يك مثلث كروي )

حالا وسط مسير ( وسط دايره عظيمه بين دو شهر ) را در نظر بگيريد و نصف النهار گذرنده از اين نقطه را هم رسم كنيد

حالا شما 2 تا مثلث كروي يكسان داريد كه بايد مختصات نقطه وسط مسير را حساب كنيد

البته استدلال شما براي طول جغرافيايي نقطه وسط مسير درست است و مي شود نصف اختلاف طول هاي جغرافيايي دو شهر

اما عرض جغرافيايي اين نقطه را بايد با استفاده از روابط مثلثات كروي بدست بياوريد

( راهنمايي : عرض جغرافيايي در نقطه وسط مسير بزرگتر از 65 درجه خواهد شد )

ali493
09-18-2012, 04:44 PM
باسلام این سوال من را کاملا گیج کرده است
پیش ستاره ای به جرم خورشید وشعاع 500 برابر خورشید با استفاده از تراکم گرانش تابش خود را ساطع کرده است اگر این فرایند 100 طول بکشد متوسط درخشندگی آن چقدر است (جرم این پیش ستاره 80 درصد هیدروژن و20 درصد هلیوم است)کتاب الفبای المپیاد نجوم فصل3 آخه کتاب شکلی این مسئله را حل کرده که نا مفهومه لطفا جواب مساله را بگذارید

المپیاد نجوم
10-10-2012, 09:24 PM
ببین به این انرژی می گن انرژی خود گرانشی که همیشه یه ضریبی از gm^2/r هست و در حالت خاصی که کره ی همگن و... باشه ضریب ما3/10 میشه!حالا شما جرم و شعاع رو داری پس انرژی خود گرانشی رو پیدا می کنی.همه ی اون رو فرض کرده که تابش شده.اگه به واحد l نگاه کنی متوجه میشی که اگه اونو در زمان ضرب کنی به تو انرژی رو میده !به این مقیاس زمانی میگن کلوین هلمهولتز.
کتاب اومده اول یه سری کار ها کرده که اصلا نیازی بهش نبوده تا آخر پیدا کردن n.بعد از اون هم فقط انرژی جنبشی رو در این فرآیند تاثیر داده در حالیکه این طوری نیست و کل انرژی مهمه!البته معمولا توی مسائل ضرایب خیلی تاثیر داده نمی شوند.
مثلا اگه سوال انقباض گزانشی مشتری رو ببینی اصلا ضریبو نمی ذاره!
به هر حال موافقم که کتاب این سوالو خوب حل نکرده!

المپیاد نجوم
10-10-2012, 09:26 PM
یه سوال!
ما توی مدل پاکر برای باد های خورشیدی اومدیم p=2nkT گذاشتیم و گفتیم میو نیمه!ولی توی اثباتی که برای این رابطه داشتیم هیچ جا میو وارد نشده بود پس این 2 چه طور وارد میشه؟
ممنون!

المپیاد نجوم
10-15-2012, 05:26 PM
یه سوال دیگه دارم:وقتی ما می خواهیم دوران رو هم در پیدا کردن زمان سقوط آزاد در تشکیل ستاره در نظر بگیریم آیا انتگرالش قابل حله؟یا من دارم اشتباه حلش می کنم ؟

shariatzadeh
10-15-2012, 06:01 PM
حل که میشه .
هیچ انتگرالی نیست که حل نشه .
باید باهاش دست و پنجه نرم کنی .
اول سرعت رو تابعی از فاصله بدست بیار (با استفاده از معادله مستقل از زمان) بعد به جای سرعت ، مشتق مکان بر حسب زمان رو بزار و معادله رو حل کن . حواست به حد های انتگرال و علامت های شتاب باشه.

المپیاد نجوم
10-15-2012, 07:45 PM
من متوجه منظورتون هستم و خودم هم همین کارو کردم ولی به انتگرالی رسیدم که خیلی ناجور شده میشه راه حلمو چک کنین که اگه راه حل مشکل داره اصلاح کنم و اگر نه کمکم کنید که انتگرالو بگیرم!
http://up.avastarco.com/images/vi7ycnwltnk0onv3w0.jpg
ممنون!

المپیاد نجوم
10-15-2012, 08:54 PM
و یک سوال دیگه.گفتین هیدروژن از هلیوم زودتر واگن میشه.توجیه این مثل مورد مربوط به الکترون و نوتورن نیست؟(سوال آقای چرتاب)
و قسمت زمان رسانش هم من درست نفهمیدم .شما میگید که زمان رسانش خیلی طولانیه و هسته همرفتیه و از طرفی میگید زمان قابل صرف نظره .البته قطعا شما درست گفتید و من نفهمیدم میشه یکم بیشتر توضیح بدید؟
ممنون!

shariatzadeh
10-16-2012, 06:39 PM
یه اشتباه کردی . تو خط اول همه جمله ها انرژی بر واحد جرم هست به غیر از جمله سوم . که خود انرژی هست . یه چیز دیگه برای چی یه لایه رو فرض کردی و ممان اینرسی رو هم در نظر گرفتی؟ باید یه ذره رو فرض کنی .
شتاب وارد به ذره رو بنویس : a=-GM/r^2+rw^2=d^2 r/ dt^2
حالا معادله دیفرانسیل رو حل کن .

المپیاد نجوم
10-16-2012, 07:05 PM
آخه تو سوال گفته بود که مدت زمانی که یک پوسته ی گرانشی تحت یک میدان با در نظر گرفتن دوران سقوط می کند را پیدا کنید.
برای همین ممان اینرسی رو هم وارد کردم.
روش شما رو متوجه شدم ولی اگر همون کار و انرژی بخواهیم بریم چی؟نمی تونیم حالتی که تازه می خواد فروبریزه و حالتی که به شعاع مورد نظرمون رسیده رو در نظر بگیریم و برای پوسته روابطو بنویسیم؟

arashgmn
10-16-2012, 07:06 PM
و یک سوال دیگه.گفتین هیدروژن از هلیوم زودتر واگن میشه.توجیه این مثل مورد مربوط به الکترون و نوتورن نیست؟(سوال آقای چرتاب)
این سوال آقای چرتاب رو من ندیدمش ! بنویس متن سوال رو تا بحث شه روش .


و قسمت زمان رسانش هم من درست نفهمیدم .شما میگید که زمان رسانش خیلی طولانیه و هسته همرفتیه و از طرفی میگید زمان قابل صرف نظره .ا
زمان رسانش در این هسته ها خیلی هم کمه !!! منظور من این بود که در کل میشه هسته رو هم دما فرض کرد . مثلا اگه یه ناحیه از هسته دماش بالاتر رفت ، چند ساعت (تا چند روز بعد)* بعد کل هسته با این دمای جدید به تعادل خواهد رسید و دوباره هم دما خواهد شد .
زمان رسانش (بهتره از واژه ی مقیاس زمانی استفاده کنیم البته !) رو بذار یه جور دیگه بگم . فرض کن یه آشفتگی دمایی در شعاع r از مرکز ستاره (و نه در محدوده ی مرکز !) روی بده . رسانش در واقع منتقل شدن انرزی توسط برخورد اتمها و مولکول ها به هم هستش. چون ستاره از گاز تشکیل شده ، پس اتم ها خیلی خیلی کم به هم برخورد میکنن و بنابراین انرژی خیلی آهسته منتقل میشه . از طرفی با مقایسه ی روش های دیگه ی انتقال انرژی(تابش و همرفت) با رسانش در یک ستاره ی عادی ، می فهمیم که عملا میشه از سهم رسانش در این انتقال ، صرف نظر کرد .
راستی تو این مثال هسته همرفتی نیست ! حداقل بعد از این که هسته ی هلیومی قابل توجهی تشکیل شد ، دیگه تو اون قسمت هلیومی (برای ستاره های پر جرم) همرفتی نیست !


حس غریبی بهم میگه که این پست رو خیلی شفاف توضیح ندادم
الان که فکر می کنم می بینم که چه قدر میشه یه موضوع رو بد توضیح داد !!! ;)

*: این مدت چند روز در مقابل عمر چندمیلیون یا میلیارد ساله ی ستاره لحظه ی خیلی کوچیکی حساب میشه .

المپیاد نجوم
10-16-2012, 08:26 PM
توی سوال 145 مجموعه سوالات آقای چرتاب گفته شده که:
با توجه به این که المان دیفرانسیلی حجم در یک فضای شش بعدی از ضرب جزء دیفرانسیل هر مولفه حاصل می شود اپبات کنید وقتی گازی را متراکم می کنید اولین ذراتی که به حالت دژنره در می آیند الکترون های آزاد هستند و بدین ترتیب می شود نتیجه گرفت چگالی ستارگان کوتوله ی سفید که در آن الکترون ها در حالت دژنره هستند کمتر از چگالی ستارگان نوترونی می باشد که در آن نوترون ها به حالت دژنره در آمده اند.
من از روابط فصل 16 مادرن استفاده کردم و نتیجه گرفتم ولی در مورد هیدروژن و هلیوم چه طور؟بازم این روش موثره؟

albertini
10-24-2012, 11:45 PM
ببخشيد دوستان
من به تازگي در مباحث نجوم كروي به موضوعي به نام z چرخان در حل مسائل برخوردم .
كسي ميتواند مختصرا توضيح دهد كه z چرخان كلا يعني چي ؟
( بيشتر فرمول ها و روابط را بلدم ولي اين اصطلاح را تا حالا نشنيدم )

albertini
10-25-2012, 11:19 PM
ببخشيد دوستان
من به تازگي در مباحث نجوم كروي به موضوعي به نام z چرخان در حل مسائل برخوردم .
كسي ميتواند مختصرا توضيح دهد كه z چرخان كلا يعني چي ؟
( بيشتر فرمول ها و روابط را بلدم ولي اين اصطلاح را تا حالا نشنيدم )

ببخشيد دوستان كسي چيزي نميدونه درباره z چرخان ؟

ميخواستم ببينم اصلا چنين چيزي هست يا يك اصطلاح اشتباهه ؟ چون در هيچ كتابي چيزي درباره اش ننوشته

M42
10-26-2012, 01:15 AM
من توی كتاب اسمارت اين اصطلاحی رو كه ميگيد نشنيدم . فكر می كنم اشتباه شنيديد و همچين چيزی وجود نداره

Astronomy
10-26-2012, 01:41 AM
ببخشيد دوستان
من به تازگي در مباحث نجوم كروي به موضوعي به نام z چرخان در حل مسائل برخوردم .
كسي ميتواند مختصرا توضيح دهد كه z چرخان كلا يعني چي ؟
( بيشتر فرمول ها و روابط را بلدم ولي اين اصطلاح را تا حالا نشنيدم )
توی یک مسئله من به یه جا بر خورده بودم که باید برای حل z رو در حال چرخیدن دور یک نقطه در نظر میگرفتیم

یعنی یه جسم داشتیم که دور سمت الراس که z بود میچرخید و برای حل شدن مسئله باید دستگاهمون رو تغییر میدادیم و اون جسم رو ثابت میگرفتیم و فرض میکردیم z دور جسم میچرخه

اینجوری مسئله حل میشد! احتمالش هست این چیزی که شما میگید هم همون باشه!!

arashgmn
10-26-2012, 11:56 AM
یه سوال دیگه دارم:وقتی ما می خواهیم دوران رو هم در پیدا کردن زمان سقوط آزاد در تشکیل ستاره در نظر بگیریم آیا انتگرالش قابل حله؟یا من دارم اشتباه حلش می کنم ؟
در حالت کلی اگه بخوایم بدون وارد کردن معادلات ترمودینامیکی مربوط به گاز ، و صرفا با توجه به دینامیک مساله اونو حل کنیم ، میشه شتاب گرانشی رو با شتاب (مجازی)گریز از مرکز همزمان برابر با جرم ذره ی آزمون در شتابش قرار داد که همونطوری که محمدجوادم گفت میشه :

a=-GM/r^2+rw^2=d^2 r/ dt^2
حالا معادله دیفرانسیل رو حل کن .
اما برای این سوال که

روش شما رو متوجه شدم ولی اگر همون کار و انرژی بخواهیم بریم چی؟نمی تونیم حالتی که تازه می خواد فروبریزه و حالتی که به شعاع مورد نظرمون رسیده رو در نظر بگیریم و برای پوسته روابطو بنویسیم؟
باید گفت : معادله دیفرانسیل بالا رو یه نگاهی بندازید دوباره . معادله یک معادله ی دیفرانسیل مرتبه دوم نسبت به زمانه . اگه همون تکنیکی رو که برای بدست آوردن انرژی از معادله ی شتاب، استفاده می کردیم ، دوباره انجام بدیم (که همون مشتق زنجیره ایه) به چی میرسیم ؟ خوب دوباره به انرژی ! اما این بار انرژی دورانی هم وارد شده . حالا می مونه حدود انتگرال ها یا به عبارت فیزیکی تر : شرایط اولیه و نهایی که تو یکی سرعت اولیه صفره و تو اون یکی ...

(یه نکته ای وجود داره در این "... " آخر! :) جالبه روش فکر کنید.)

المپیاد نجوم
10-26-2012, 05:49 PM
z چرخان توی بعضی مسائل کاربرد داره.
تو این روش میایم و حالت واقعی رو درنظر می گیریم یعنی ما متحرکیم و ستارگان ثابت هستند.
پس zکه سرسویمان هست را این بار متحرک می گیریم که روی یک دایره صغیره حرکت می کند.البته اگر عرض جغرافیایی صفر باشد دایره صغیره تبدیل به عظیمه می شود.
حالا توی مسائلی مثل سوال مرحله دوم دوره 6 (سوال 4)شما باید z رو چرخون بذاری و دو تا شرط رو پیدا کنی و مسالتو حل کنی!

المپیاد نجوم
10-26-2012, 05:50 PM
در مورد سوال خودم !آقای گل محمدی میشه لطفا علت این که نباید پوسته بگیرم رو بهم بگید؟
منظورتون از ... سرعت نهایی هست؟

arashgmn
10-26-2012, 11:13 PM
در مورد سوال خودم !آقای گل محمدی میشه لطفا علت این که نباید پوسته بگیرم رو بهم بگید؟
منظورتون از ... سرعت نهایی هست؟

من جدا یه پیشنهادی به همه می کنم . چیزی که خیلی به من کمک کرد. وقتی یه سری مسائل رو می خواید حل کنید ، سعی کنید اول اونو در نظر بگیرید و حـــدس بــزنیـــد که چه بلایی سرش میاد! تو مواردی که مربوط به فیزیک کلاسیک میشه ، معمولا دید خوبی به آدم میده .
اما ربطش به سوال( تمامی * هایی که در نوشته هستن در آخر توضیح داده شدن . سعی نکنید همون لحظه بخونیدشون و بذارید آخر کار!) :
سوال گفته
که مدت زمانی که یک پوسته ی گرانشی تحت یک میدان با در نظر گرفتن دوران سقوط می کند را پیدا کنید.
خوب شما اگه بخواید حدس بزنید چی میشه ،اول باید تصورش کنید . یه ابر کروی رو (که احتمالا از ذرات تک اتمی و خنثی تشکیل شده) در نظر میگیریم . طبق گفته ی سوال باید دوران هم تاثیر بدیم . پس این ابر کروی مون در لحظه ی اول داشته بایه سرعت زاویه ای اولیه ی اومگا صفری می چرخیده * .
خوب این ابره بر اثر گرانشش منقبض میشه و این انقباض همین طوری سرعت می گیره ... می گیره ؟ ...؟ پس دوران چی؟
با کوچیک شدن ابر ، احتمال میدیم که ابر تند تر بچرخه و این یعنی یه نیروی مرکز گریز! پس ممکنه که انقباض سرعتش کند شه ... ؟! یا حتی متوقف بشه و شروع به انبساط کنه !!!

خوب از این جا به بعد، "شهود" چندتا احتمال رو برامون مشخص میکنه.میشه بازم سرنوشت هر کدوم از این احتمالا رو باز هم حدس زد اما خیلی از سوال دور میشیم . در ضمن می دونیم که طی شدن یکی از این احتمال ها ، احتمالا به شرایط اولیه بستگی داره .پس نمیشه با قطعیت پیش از نگارش چیزی بهشون پاسخ داد. این جا همون جاییه که دیگه باید دست به قلم شد و معادلات رو نوشت !

(با قبول یک فرض**...: ) خوب می تونیم معادله ی شتاب*** یک عنصر جرم پوسته ای رو بنویسیم اما برای تمام پوسته نیروی مرکز گریز یکسان نیست ( قطب دوران اصلا چنین نیرویی را حس نمی کند!) . پس خودمون رو به یه حالت خاص محدود می کنیم. یه عنصر جرم حلقوی روی یک عرض جغرافیایی خاص! و چه جایی بهتر از استوا؟(گرچه برای بقیه عرض ها فقط یک جمله ی کسینوس عرض ظاهر می شه و جواب تغییری نمی کنه.) اگه معادلات شتاب رو بنویسیم و ساده کنیم ، می بینیم که معادله ی ساده شده در واقع معادله ی حرکت یه ذره رو استوا رو بهمون میده ( که البته از تقارن واضح هم بود که چنین باید بشه).
بعدشم شروع می کنیم به حل معادلات !

*: کی گفته ابر اولیه همش با سرعت زاویه ای یکسانی می چرخیده؟ حالا اگه به فرض تو لحظه ی اول این جوری بوده ، چه تضمینی هست که بعدشم این جوری بمونه ؟ مگه با یه جسم صلب طرفیم ؟ نمونه اش هم همین خورشید و مشتری ...
** : از اون جایی که هدفمون حل مسئله اس ، مجبوریم یه سری آرمانی سازی انجام بدیم . پس فرض می کنیم سرعت زاویه ای در تمام مراحل برای کل ابر یکسانه.
***:این که تاکید میشه معادله ی شتاب رو بنویسید به این علته که معادله شتاب ، بنیادین تر از معادله ی انرژیه. هرچند که برای حلش باید همانند انرژی ، زنجیره ای بزنید و به خود انرژی برسید...

پ.ن: اون ... در واقع حاصل حدس زدن ماست ! در بالا به یک دو راهی برخورد کردیم : این که انقباض متوقف می شود یا نه ! خوب این در چی خودشو نشون میده؟ معلومه !در سرعت !

منظورتون از ... سرعت نهایی هست؟ بله !

MOON
11-17-2012, 11:50 PM
آيا شار نوراني با روشنايي (b) تفاوت داره يا يكي هست ؟

واحد هر دو وات بر متر مربعه ؟

و آيا مجموع شار نوراني يك ستاره در فاصله يكسان برابر با شار تابشي اون ميشه يا نه ؟

مرسي

المپیاد نجوم
11-18-2012, 12:07 AM
تا جایی که من می دونم واحد هر دو تا وات بر متر مربعه ولی این دوتا فرق می کنن.
شار نورانی یعنی اون انرژی در واحد زمان که به واحد سطح جسمی توی فاصله ی dمی رسه ولی روشنایی یعنی باز همون انرژی در واحد زمانی که روی خود سطح از دید ناظر توی فاصله ی dاندازه گرفته میشه(یه جورایی انگار ذاتی تره!)
یه چیزی
شار تابشی مگه همون شار نورانی نیس؟؟؟؟

MOON
11-18-2012, 12:16 AM
بله
منم تو بعضي منابع ديدم شار نوراني و روشنايي رو يكي مي گيرن و بعضي جاها ديدم فرق ميذارن
ولي واحد هر دو هم وات بر متر مربعه
ولي شار تابشي با اينها فرق داره
شار تابشي ميشه مقدار انرژي كه در واحد سطح و واحد زمان از سطح ستاره تابش ميشه
و از رابطه زيگما x دما به توان 4 به دست مياد
ولي واحد اون هم وات بر متر مربعه

المپیاد نجوم
11-18-2012, 12:23 AM
آره دیگه این همون ماجرای تعاریفه !
یعنی طبق این تعریف شار تابشی روی سطح خود ستارست و شار نورانی یا همون روشنایی(طبق این تعریف)اونیه که به ما می رسه !

المپیاد نجوم
11-28-2012, 07:36 PM
من اومدم با یه عالمه سوال!
1.در مورد سوالای آخر جزوه ی عناصر مدار .
چون سوال 4 با راه حلی که به ذهن من رسید طولان میشه می خواستم اول راه حلمو بگم که اگه درست بود محاسبات رو بنویسم!
من گفتم چون حرف از زمان زده احتمالا باید از معادله کپلر استفاده کنیم.یعنی اول آنومالی mرو پیدا کنیم و بعد هم eو بعد هم تتای جدید رو.از طرفی چون بردار سرعت و مکان رو داریم همه چیز در مورد مدار در میاد.تنها چیزی که عوض میشه هم تتاست که اونم در آوردیم.حالا واسه این که چن معادله چن مجهولو حل کنیم فکر کردم شاید بد نباشه از پایستگی تکانه زاویه ای و روابط خروج از مرکز با توجه به پابت بودنش استقاده کنیم.درسته؟؟؟راه کوتاه تری هم این مساله داره؟
2.من راه حل خیلی خوبی برای سوال 2 همون جزوه ی عناصر مدار به ذهنم نرسید.میشه یکم راهنماییم کنین؟
3.اگه 4 تا مورچه روی یک مستطیل راه بروند.(سوال مشهوره رو میگم!)مسیر حرکت چی میشه؟(فقط چک می خوام بکنم.)
4.من توی مبحث گرانش خیلی لنگ می زنم!توی سوالات آخر فصل 4 ماریون هم سوال 8و9و10و11و14 رو نتونستم حل کنم.چه منبعی هست که کامل تر گرانش رو ازش بخونم؟
ممنون!

shariatzadeh
11-28-2012, 08:51 PM
درباره سوال 4 :
شما در این مسئله چندین راه برای رسیدن به جواب دارید اما شاید در اولین نگاه راه زیر به ذهنتون برسه .
با توجه به مطالبی که در طول این جزوه گفته شده با استفاده از بردارهای مکان و سرعت ابتدا عناصر مداری را بدست می آوریم بعد همانطور که گفته شد با استفاده از معادله کپلر تتای جدید رومحاسبه می کنیم . حالا مسئله محاسبه بردار مکان و سرعت جدید با استفاده از عناصر مدار هست که نمونه اش در جزوه حل شده است .
اما یک راه ساده تر وجود دارد که به زودی در تاپیک مکانیک مداری می نویسم .

درباره سوال 2 :
صفحه 6 جزوه معادله ای برای بدست آوردن بردار خروج از مرکز وجود دارد . باید از این معادله استفاده کنید . و یک راهنمایی دیگه اینکه زاویه بین بردار r و e برابر با یک پارامتر آشنا است که شما باید ایتدا اون رو بدست بیارید .

3- توجه کنید که هر مورچه همیشه به سمت مورچه مجاور خود حرکت می کند .
شما باید نشان دهید که اگر در هر لحظه مورچه ها را به هم وصل کنیم ، شکلحاصل مستطیلی است که با مستطیل اولیه متشابه است و مقداری چرخیده . اون وقت نتیجه میشه که بردار مکان و سرعت مورچه ها نسبت به مرکز مستطیل باید مقدار ثابتی باشه و بعد جواب یک منحنی نمایی خواهد بود . (چی گفتم ؟!!)

4- یه جوری میگی نمیتونم تمرین های آخر ماریون رو حل کنم انگار ماریون کتاب فیزیک دبیرستانه و همه صد بار همه تمرین هاش رو حل کردن . باید تلاش کنی ، ماریون خیلی سطحش بالا و ریاضیت باید خوب باشه تا بتونی سوالاش رو حل کنی . نگران نباش .

Amir Shayan Nejati
12-08-2012, 07:55 PM
سلام من مفهوم نیروی مجازی روخوب نفهمیدم ویه سوال داشتم...
توی این سوال معروفی که میگه مایک سطل روباسرعت زاویه ای امگا میچرخونیم بعد بایدثابت کنیم شکل بالای آب سهمی میشه این سوال روکلپنرتوفصل8 گفته که نیروی مجازیش mrw^2هست.ولی یکی از استادها کلا اصلا این رونگفت.حالا سوال من اینه:
چرانیروی مجازی رو وارداین سوال میکنیم؟؟نمیتونیم بگیم چون داره بایک زاویه تتامیچرخه پس این ترم نیرورو قطبی مینویسیم؟؟

erfan bayat
12-08-2012, 10:07 PM
سلام من مفهوم نیروی مجازی روخوب نفهمیدم ویه سوال داشتم...
توی این سوال معروفی که میگه مایک سطل روباسرعت زاویه ای امگا میچرخونیم بعد بایدثابت کنیم شکل بالای آب سهمی میشه این سوال روکلپنرتوفصل8 گفته که نیروی مجازیش mrw^2هست.ولی یکی از استادها کلا اصلا این رونگفت.حالا سوال من اینه:
چرانیروی مجازی رو وارداین سوال میکنیم؟؟نمیتونیم بگیم چون داره بایک زاویه تتامیچرخه پس این ترم نیرورو قطبی مینویسیم؟؟
ببین اگه اثبات شتاب در دستگاه قطبی دیده باشی دو مولفه ی شعاعی و تتا داره.پس هر نیرویی حقیقی در راستای شعاعی برابر مولفه ی شعاعی شتاب هستش و در راستای تتا هم همینطور.
برای این مسئله دوتا نیروی حقیقی داریم.1- نیروی گرانش 2- نیروی تکیه گاه که عمود بر سطح آبه که یه زاویه تتا با راستای نیروی گرانش فاصله دارد.
خوب در راستای شعاعی مشتق 2 شعاع صفره چون حرکت دایره ای و فقط مولفه ی mrw^2- باقی میمونه که برابر نیروی تکیه گاه در سینوس تتا
در راستای مماسی یا همون تتا طبق جمله ی بالا دوتا مولفه صفرند پس نیروهای حقیقی mg=Ncosx که اگه این دوتا رو به هم تقسیم کنی تانژانت تتا برابر مشتق Y به مشتق r که با یه انتگرال ساده سهمی میشه.
خلاصه منظور اینه همیشه اصولی مسئله رو حل کن. اصلا در مورده نیرو مجازی فکر نکن بزار تو رابطه شتاب و با توجه به شرایط جوابو به دست بیار

arashgmn
12-10-2012, 09:48 PM
من فصل 16 رو نگاه کردم . کلی رابطه داره . نفهمیدم که منظورت کدومه . میشه راه حلت رو بنویسی ؟( حدالامکان رو کاغذ بنویس و بعد آپلود کن که هم نوشتنش و هم خوندنش راحت تر باشه.)

هانیه کاشانی
12-29-2012, 02:37 PM
سلام

من دانش آموز سال دوم دبیرستان سلام سبز هستم راستش مشاور مدرسه ی ما به ما گفتند اگر پیشرفت چشم گیری در درسهایمان داشته باشیم برایمان معلم میگیرند

میخواستم بپرسم آیا تدریس در مدرسه برای کسی از اعضای اینجا امکان پذیر هست یا نه ؟

Astronomy
12-29-2012, 02:56 PM
سلام

من دانش آموز سال دوم دبیرستان سلام سبز هستم راستش مشاور مدرسه ی ما به ما گفتند اگر پیشرفت چشم گیری در درسهایمان داشته باشیم برایمان معلم میگیرند

میخواستم بپرسم آیا تدریس در مدرسه برای کسی از اعضای اینجا امکان پذیر هست یا نه ؟
سلام

بله تعداد زیادی از مدال آوران المپیاد نجوم در این فروم هستند، از جمله:

Ehsan (http://forum.avastarco.com/forum/member.php?545-Ehsan)، Astronomer (http://forum.avastarco.com/forum/member.php?251-Astronomer)، shariatzadeh (http://forum.avastarco.com/forum/member.php?2792-shariatzadeh)، erfan bayat (http://forum.avastarco.com/forum/member.php?1783-erfan%20bayat)، ata moradi (http://forum.avastarco.com/forum/member.php?1840-ata-moradi)، arashgmn (http://forum.avastarco.com/forum/member.php?3538-arashgmn)، پیمان اکبرنیا (http://forum.avastarco.com/forum/member.php?u=1120)، و تعدادی دیگر از دوستان

Negin_GH
12-29-2012, 03:33 PM
سلام:)

میشه لطفا بگین فرق مرحله اول و دوم توی نوع سوالا چیه؟؟منابع این دو مرحله با هم فرق میکنه؟و اینکه برا مرحله اول چه کتابایی باید بخونیم؟؟
اگه سوالم تکراری یا مسخره بود یا جاش اینجا نبود ببخشید.در کل ببخشید:):گل

Zareie
12-31-2012, 11:50 AM
سلام به همه
ببخشید ، میشه من رو در مورد حل این مسئله کمک کنید:

فاصله دو برج به ارتفاع 435 متر حداقل چقدر باشد تا افرادي كه در فراز انها قرار دارند يكديگر را نبينند؟

پیمان اکبرنیا
12-31-2012, 06:20 PM
سلام:)

میشه لطفا بگین فرق مرحله اول و دوم توی نوع سوالا چیه؟؟منابع این دو مرحله با هم فرق میکنه؟و اینکه برا مرحله اول چه کتابایی باید بخونیم؟؟
اگه سوالم تکراری یا مسخره بود یا جاش اینجا نبود ببخشید.در کل ببخشید:):گل

سلام به شما

موضوعاتی که در این دو مرحله می آیند تقریبا شبیه هستند ولی سوالات مرحله دوم سطح بالاتری دارند و باید مهارت حل مساله بیشتری داشته باشید. برای مرحله اول اگر می خواهید امسال شرکت کنید و هنوز چیزی نخوندید تنها راهی که سریع جواب میده اینه که کتاب حل سوالات سالهای قبل را بگیرید و یاد بگیرید. چون وقت زیادی برای خوندن کتابهای پایه ندارید و باید سریع فقط یاد بگیرید چگونه سوالات مشابه سالهای قبل را حل کنید (به اصطلاح تزریق فشرده مطالب)

پیمان اکبرنیا
12-31-2012, 06:27 PM
سلام به همه
ببخشید ، میشه من رو در مورد حل این مسئله کمک کنید:

فاصله دو برج به ارتفاع 435 متر حداقل چقدر باشد تا افرادي كه در فراز انها قرار دارند يكديگر را نبينند؟

شما باید به این فکر کنید که اگر بالای یک برج باشید تا چه فاصله ای را میتوانید روی سطح زمین ببینید. یک دایره بکشید با یک برج روی آن. از بالای برج دو خط بر دایره مماس کنید. فاصله بین دو نقطه مماس میشه مناطقی که از بالای برج معلومه :) حالا با داشتن ارتفاع برج و شعاع زمین ببینید میتونید حساب کنید تا چه فاصله ای دیده میشه؟ باز هم اگر راهنمایی خواستید یک مرحله جلوتر را خواهم گفت :)

celestial boy
12-31-2012, 06:27 PM
سلام به همه
ببخشید ، میشه من رو در مورد حل این مسئله کمک کنید:

فاصله دو برج به ارتفاع 435 متر حداقل چقدر باشد تا افرادي كه در فراز انها قرار دارند يكديگر را نبينند؟
سلام.
یه مقدار حدی اینجوری پیدا میشه.دوتا برجو طوری می کشی که خط واصل دو سرشون به زمین مماس باشه.حالا دوتا مثلث قائم داریم که با حلشون زاویه بین دو برج به دست میاد.و بعدشم فاصله بین دو برج. اینم شکلش.
http://up.avastarco.com/images/awueubd3wudrc3illgq2.jpg

(پست بنده با آقای اکبرنیا همزمان شد)

المپیاد نجوم
01-06-2013, 06:06 PM
یه سوال دیگه داشتم.
توی مجموعه سوالات آقای چرتاب سلطانی سوال 30 من تابعیت ارتفاع رو به دست آوردم که هم تابع تغییرات عرض جغرافیایی میشه و هم زمان.ولی برای حل قسمت های بعد هر کاری می کنم حس می کنم یه داده(سمت ستاره)کمه!نمی دونم اشتباهم کجاس میشه راهنماییم کنین؟
ممنون!

ESA Moon
01-06-2013, 06:24 PM
فقط خواستم بگم مهلت ثبت نام تا چهارشنبه این هفته هم تمدید شد!
دیگه از خدا چی می خوای برو حلشو ببر

پیمان اکبرنیا
01-06-2013, 11:56 PM
یه سوال دیگه داشتم.
توی مجموعه سوالات آقای چرتاب سلطانی سوال 30 من تابعیت ارتفاع رو به دست آوردم که هم تابع تغییرات عرض جغرافیایی میشه و هم زمان.ولی برای حل قسمت های بعد هر کاری می کنم حس می کنم یه داده(سمت ستاره)کمه!نمی دونم اشتباهم کجاس میشه راهنماییم کنین؟
ممنون!

سلام :)

ببخشید میشه لینک دانلود سوالات یا تصویری از سوال هم بگذارید که کسانی که دسترسی ندارند هم ببینند و بتونند مشارکت کنند؟ ;)

المپیاد نجوم
01-07-2013, 01:05 PM
http://www.4shared.com/document/rRlMOLPu/nchsoltani.html
این لینک دانلود سوالات!سوال 30 رو اگه میشه راهنمایی کنید.
ممنون!

المپیاد نجوم
01-07-2013, 01:05 PM
http://www.4shared.com/document/rRlMOLPu/nchsoltani.html
این لینک دانلود سوالات!سوال 30 رو اگه میشه راهنمایی کنید.
ممنون!

celestial boy
01-07-2013, 02:33 PM
سلام.
با این که نمی تونم حلش کنم.ولی فک کنم سمت ستاره رو داشته باشیم.اونجایی که گفته داره به سمت ستاره حرکت می کنه.اگه بشه گفت سمت ستاره واسش تغییر نمی کنه احتمالا بشه اونو زاویه مثلث کروی (قطب-تبریز-تهران) توی راس تهران در نظر گرفت.چی گفتم!!!!!!!!؟

المپیاد نجوم
01-08-2013, 03:51 PM
خب اون فرض مساله نیست که سمت براش ثابته!اگر اون طوری بگیم خوب ساده میشه تقریبا!
حالا به جز اون مساله من یه سوال دیگه هم دارم.توی یکی از سوالات جهانی در مورد تابش بند گفته بود.میشه بگین تابش بند چیه و چه کار می کنه و کلا محاسباتش چه طوریه؟
ممنون!

پیمان اکبرنیا
01-09-2013, 09:40 PM
یه سوال دیگه داشتم.
توی مجموعه سوالات آقای چرتاب سلطانی سوال 30 من تابعیت ارتفاع رو به دست آوردم که هم تابع تغییرات عرض جغرافیایی میشه و هم زمان.ولی برای حل قسمت های بعد هر کاری می کنم حس می کنم یه داده(سمت ستاره)کمه!نمی دونم اشتباهم کجاس میشه راهنماییم کنین؟
ممنون!

در این سوال فکر کنم باید فرض کنید طول و عرض جغرافیایی تهران و تبریز را دارید. اگر این داده ها را داشته باشید معلوم میشه تبریز از دید تهران در چه سمتی قرار داره (یعنی در افق تهران زاویه تبریز با سمت شمال جغرافیایی چقدره). سمت ستاره هم در لحظه شروع حرکت همون میشه چون ناظر داره داره در ابتدا به سمت ستاره حرکت میکنه. فکر کنم با این فرض بتونید حل کنید نه؟